<<

Introduction to and Computation

Steven M. Girvin ⃝c 2019, 2020

[Compiled: May 2, 2020] Contents

1 Introduction 1 1.1 Two-Slit Experiment, Interference and Measurements . . . . 2 1.2 Bits and ...... 3 1.3 Stern-Gerlach experiment: the first ...... 8

2 Introduction to 16 2.1 Linear Operators on Hilbert Space ...... 18 2.2 Dirac Notation for Operators ...... 24 2.3 Orthonormal bases for electron states ...... 25 2.4 Rotations in Hilbert Space ...... 29 2.5 Hilbert Space and Operators for Multiple Spins ...... 38

3 Two-Qubit Gates and Entanglement 43 3.1 Introduction ...... 43 3.2 The CNOT Gate ...... 44 3.3 Bell Inequalities ...... 51 3.4 Quantum Dense Coding ...... 55 3.5 No-Cloning Theorem Revisited ...... 59 3.6 ...... 60 3.7 YET TO DO: ...... 61

4 Quantum Error Correction 63 4.1 An advanced topic for the experts ...... 68

5 Yet To Do 71

i Chapter 1

Introduction

By 1895 there seemed to be nothing left to do in physics except fill out a few details. Maxwell had unified electriciy, magnetism and optics with his theory of electromagnetic waves. Thermodynamics was hugely successful well before there was any deep understanding about the properties of atoms (or even certainty about their existence!) could make accurate predictions about the efficiency of the steam engines powering the industrial revolution. By 1895, statistical mechanics was well on its way to providing a microscopic basis in terms of the random motions of atoms to explain the macroscopic predictions of thermodynamics. However over the next decade, a few careful observers (e.g. Planck and especially Einstein) began to realize that the very foundations were crum- bling. Maxwell correctly taught us that accelerating charges radiate away energy in the form of electromagnetic waves. Why then do electrons orbit- ing the positively charged nuclei in atoms not radiate away all their energy and fall into the nucleus? This doesn’t happen, and in fact, physicists soon discovered that the lone electron in the hydrogen atom seems to have only a discrete set of allowed orbits corresponding to certain well-defined ener- gies. Electrons could in fact emit electromagnetic waves, but only in discrete lumps of energy (‘quanta’ known as ‘photons’) that matched the change in energy as the electron spontaneously jumped from one allowed to a lower allowed energy level. Once the electron reached the lowest allowed energy level (the ‘’), it could fall no further despite the fact that it was still accelerating while orbiting the nucleus). Maxwell also taught us that the modes of the electromagnetic field are harmonic oscillators. Why then does electromagnetic radiation not obey the

1 CHAPTER 1. INTRODUCTION 2 laws of statistical mechanics developed for harmonic oscillators? Accord- ing to classical statistical mechanics, the average energy stored in the kinetic and potential energy of a harmonic oscillator is kBT , where kB is Boltzmann’s constant and T is the absolute temperature. Since electromagnetic radiation comes in a wide range of frequencies running all the way up to infinity, clas- sical statistical mechanics predicts an ‘ultraviolet catastrophe,’ in which the energy density of blackbody radiation (i.e. radiation in thermal equilibrium) should be kBT × ∞. Even venerable thermodynamics, which makes predictions of a general nature independent of microscopic details, gave some hints of problems. The thermodynamic entropy (a measure of our ignorance of the microscopic de- tails of the state of the system; or equivalent a measure (the logarithm) of the number of different microscopic states that are consisten with the observed macroscopic properties) of a gas obeying the ideal gas law turns negative at low temperatures. In statistical physics, Josiah Willard Gibbs pointed out the now famous ‘Gibbs paradox.’ He noted that if a partition is removed from a box allowing the gas atoms on each side to mix, the entropy should irreversibly increase, and yet nothing macroscopically actually happens. The classical way of counting up the number of different configu- rations of a bunch of atoms was somehow wrong. The Gibbs paradox could be resolved only if exchanging the positions of two particles (say particle 17 and particle 1,230) did not result in a new state–as if the atoms were not merely the same species but truly indistinguishable.

1.1 Two-Slit Experiment, Interference and Measurements

TO BE WRITTEN: Wave-like interference and . Particle-like detection. Which path information and state collapse upon mea- surement. CHAPTER 1. INTRODUCTION 3 1.2 Bits and Qubits

A bit is the smallest unit of information. The word ‘bit’ is short for binary digit and can be represented by 0 or 1 in the binary numbering system (base 2 numbers). It is the amount of information contained in the answer to a yes/no or true/false question (assuming you have no prior knowledge of the answer and the two answers are equally likely as far as you know). If Alice gives Bob either a 0 or a 1 drawn randomly with equal probability, then Bob receives one bit of information.1 A register of N classical bits can be in one of 2N distinct states. For example 3 classical bits can be in 8 states: (000), (001), (010), (011), (100), (101), (110), (111), corresponding to the 8 binary numbers whose (base 10) values run from 0 to 7. This is a remarkably efficient encoding–with an ‘alphabet’ of only two symbols (0 and 1) Alice can send Bob one of 232 = 4, 294, 967, 296 possible messages at a cost of only having to transmit 32 bits (or 4 ‘bytes’). In fact, if all the messages are equally likely to be chosen, this is provably the most efficient possible encoding. Information is physical and is stored in the state of a physical system. In an ordinary computer, bits are stored in the states of switches (typically these days transistors) which can be open/closed or on/off. Information can also be stored in small domains of magnetism in disk drives. Each magnetic domain acts like a small bar magnet whose magnetic moment (a vector quantity pointing in the direction running from the south pole to the north pole of the bar magnet) can pnly point up or down. Ordinarily a bar magnet can point in any direction in space, but the material properties of the disk drive are intentionally chosen to be anisotropic so that only two directions are possible. A quantum bit (‘qubit’) is information stored in a quantum system that can be in one of two possible states. Using a notation devel- oped by Paul A.M. Dirac, state |0⟩ might denote an atom in its lowest energy state (the ground state) and state |1⟩ might denote the first excited state of

1Information is ‘news’. If Bob knew ahead of time that Alice would chose 0 with probability p0 = 0.999, and 1 with probability p1 = 1 − p0, then he is not surprised when he receives a 0 because he already was pretty sure he ‘knew the answer to the question’ ahead of time. The mathematical theory of information quantifies the information that Bob receives and shows that it is much less than one bit of information. Claude Shannon demonstrated that in this situation Bob receives information H = − [p0 ln2 p0 + p1 ln2 p1], 1 where ln2 means log base 2. H has a maximum value of 1 at p0 = p1 = 2 and approaches zero when either of the probabilities approaches unity. CHAPTER 1. INTRODUCTION 4 the atom (or some other selected excited state). However, quantum systems behave strangely–particles can act like waves and waves sometimes act like particles. Two-state systems have wave-like properties in the sense that can be in superposition states represented mathematically by |ψ⟩ = α|0⟩ + β|1⟩ (1.1) where α and β are (complex) probability amplitudes. If one measures whether the qubit has value 0 or 1, the answer is random. The measure- ment yields 0 with probability2 |α|2 and 1 with probability |β|2. Thus we have the important constraint that the probabilities must add up to unity |α|2 + |β|2 = 1. (1.2) We say that the state must be ‘normalized.’ Furthermore, we see that an overall ‘global phase’ of the form eiχ|ψ⟩ = eiχα|0⟩ + eiχβ|1⟩ (1.3) (where χ is an arbitrary real number) can have no measurable effect because the probabilities for different measurement results depend only on the magni- tude square of the state coefficients. Hence, without loss of generality we can choose the phase angle χ to make α real. The β = a + ib is parametrized by two real numbers a and b. However the normalization constraint removes one of these degrees of freedom. In the end we are left with two real parameters to specify the state of a single qubit. A standard parametrization in terms of two angles θ and φ is the following θ θ |ψ⟩ = cos |1⟩ + eiφ sin |0⟩. (1.4) 2 2 The two angles θ and ϕ can be visualized as the polar and azimuthal angles of a vector pointing from the origin to the surface of the unit sphere as illus- tragted in Fig. 1.1. We will later describe the meaning of the unit vector on the Bloch sphere in relation to the properties of the corresponding quantum state and will also explain why half angles appear in the parametrization of the state in Eq. (1.4). We simply note here that the two states of a classical bit are represented by the Bloch sphere vectors corresponding to the north pole (0, 0, 1) and the south pole (0, 0, −1). Quantum bits can be in states corresponding to an arbitrary point on the sphere. 2We are using the following standard notation. If we have a complex number z = x+iy, we define complex conjugation as z∗ = x − iy and the magnitude (squared) of the number as |z|2 = z∗z = x2 + y2. CHAPTER 1. INTRODUCTION 5

|1² z 1 >| 0 ² |1 ² @ 1 2 >i |0 ² |1 ² @ y 2 x T

M x  y 1 1 >| 0² |1 ² @ z >i | 0² |1 ² @ 2 2 | 0²

Figure 1.1: Unit vector corresponding to a point on the Bloch sphere with cartesian coordinates (sin θ cos φ, sin θ sin φ, cos θ). The orientation of this 3D unit vector can be obtained by starting with the unit vector pointing to the ‘north pole’ of the sphere and then rotating it in the xz plane by an angle θ around the y axis and then rotating by an angle φ around the z axis. This unit vector corresponds to the parametrization of the qubit state given in Eq. (1.4).

Box 1.1. A word about notation. Since |0⟩ and |1⟩ might represent the ground and first excited states of a quantum system, we will sometimes denote them |g⟩ and |e⟩ respectively. Similarly, we may use the orientation on the Bloch sphere to label the same states | ↑⟩ and | ↓⟩. The states corresponding to (1, 0, 0) on the Bloch sphere might be written 1 1 | →⟩ = | + X⟩ = √ [|0⟩ + |1⟩] = √ [| ↓⟩ + | ↑⟩]. 2 2 It is a weird feature of quantum spins that a coherent superpostion of up and down points sideways!

It is very important to understand that the randomness in quantum me- chanics does not arise from our lack of knowledge of all the variables in the experimental set up. It is not true that the qubit has a certain value deter- mined by some ‘hidden’ variable that you do not know about. You can pre- pare two qubits in exactly the same state and still get different measurement results! In fact, the qubit does not have a ‘value’ before it is measured. We will see later that assuming that the qubit has a value before it is measured leads to contradiction with experiment. One can actually prove experimen- tally that there are no hidden variables. This leads to the following clever CHAPTER 1. INTRODUCTION 6 statement (due to Alexander Korotkov): ‘In you do not see what you get, you get what you see.’ Curiously, if (using an idealized apparatus) you measure the same qubit again, you will get the same value for the second (and all subsequent) mea- surement results. This means that the first measurement has ‘collapsed’ the state. If you start with the state α|0⟩ + β|1⟩ and the first measurement re- sult happens to be 1, then the state collapses to |1⟩ and stays there for all subsequent measurements. Conversely, if the first measurement happens to yield the value 0, then the state collapses to |0⟩ and stays there. The above results illustrate a curious asymmetry in . The state of a qubit requires specification of two real numbers (the Bloch sphere angles, θ and φ). In principle this means it takes an infinite number of bits to specify the state. And yet, when we make a measurement, the state collapses to |0⟩ or |1⟩ and we learn only one bit of information. This asymmetry has far-reaching implications as we will see later. In particular it will play an important role in the no-cloning theorem (which tells us that unknown quantum states cannot be copied). This in turn is the basis of . One might wonder why one would want to build a computer out of qubits whose measured values seem to be random. The answer (somehow) partly lies in the ability of a collection of qubits to explore a huge number of configura- tions simulataneously. Thus a collection of 3 qubits can be in a simultaneous superposition of 23 states of the form

|ψ⟩ = α0|000⟩ + α1|001⟩ + ... + α7|111⟩. (1.5) Thus if we build a quantum computer with an input register of N qubits, we can give the computer a single giant superposition of all the possible inputs we could ever give it. Because the Schr¨odingerequation describing the time evolution of the quantum state of the ocmputer is linear, the computer will carry out is calculation on every possible input simultaneously without difficulty. Thus we have what appears to be exponentially powerful ‘quantum parallelism’ in which the computer carries out 2N computations at once. Unfortunately, the output register contains a huge superposition of all 2N answers (results of the calculation). When we measure the output register the state randomly collapses to only one of the 2N answers and it seems we lose all the quantum advantage of the parallelism. It turns out however that all is not necessarily lost. There exist certain classes of problems which have simple answers and one can use the wave- CHAPTER 1. INTRODUCTION 7 like properties of the superpositions to yield constructive interference which ‘focuses’ the probability amplitudes in the output register onto the desired answer. The classic example of this is the Shor algorithm for finding the prime factors of a composite number. The input to the quantum computer is very simple–the single number to be factored. The output is also very simple–one of the factors. The algorithm almost perfectly focuses the output amplitude onto one of the factors so that measurement of the output register gives the correct answer with relatively high probability. Since factoring is a ‘one-way problem’ that is hard to solve but the solution is easy to check, one simply runs the algorithm a small number of times and checks the answer each time. Within a relatively small number of tries, you will be able to verify that you have found a factor. In this case, where the inputs and outputs are relatively simple, the power of the quantum computer (presumably) comes from its ability to explore an exponentially large number of intermediate states during the course of the calculation. The Shor algorithm is exponentially faster than the best known classical algorithm for factoring. (Factoring is believed to be exponentially hard clas- sically, but this has not actually been rigorously proven.) Other algorithms offer only polynomial speed up. For example the Grover search√ algorithm can find a given entry in an unordered data base of size N in about N queries to the (quantum) database. Consider for example a telephone directory which has the entries in random rather than alphabetically order. To find your friend’s name in this database classcially would require examining on aver- age N/2 out of the N total entries. For this problem, the quantum algorithm is not able to focus all the output amplitude onto the desired answer, but rather can partially√ focus the output probability amplitudes so that they are spread over only N output states, rather√ than all N. Thus measurement of the output register has probability p ∼ 1/ N of collapsing onto√ the desired state. One thus expects to find the correct answer in about N attempts rather than N/2 attempts as in the classical case. This is not exponential speed up but it is still interesting (and surprising) quantum advantage. The source of the power of quantum computers lies partly in the existence of superpositions but also in another subtle concept called entanglement that gives qubits ‘spooky’ correlations that can be stronger than is allowed for classical bits. We will look into this mystery in a later section. Before delving into the mathematical description of these two-state sys- tems and the information they contain, it is useful to start with experiment. CHAPTER 1. INTRODUCTION 8 1.3 Stern-Gerlach experiment: the first qubit

The first qubit was a silver atom. In Frankfurt, Germany in February 1922, physicists Otto Stern and Walther Gerlach carried out an experiment on a beam of silver atoms that had momentous import for the development of quantum mechanics and in fact constituted the first measurement of the state of a qubit. [See Physics Today article in the files section on Canvas.] They evaporated atoms from a bar of silver placed in an oven and sent them through a magnetic field that had a strong gradient as illustrated in Fig. 1.2. Stern and Gerlach did not realize it at the time, but it turns out that their experiment showed that electrons carry intrinsic angular momentum called ‘spin.’ Thinking classically we can imagine the electron as a small spherical ball of charge spinning around some axis. The resulting current flow creates a magnetic dipole–the electron acts like a little bar magnet. The magnetic moment is a vector whose direction runs from the south pole to the north pole of the bar magnet and whose magnitude is proportional to the strength of the magnetic field produced by the bar magnet. Because electrons have negative charge (thank you, Mr. Franklin), the magnetic moment vector is antiparallel to the angular momentum vector of the spinning electron. This classical argument cannot really be applied to the quantum domain where the electron is a point particle that acts like a wave. However Dirac showed that his relativistic wave equation for the electron very naturally predicted the existence of the electron spin and predicted the electron magnetic moment nearly exactly e g ⃗µ = − S,⃗ (1.6) me 2 ⃗ where me is the electron mass, −e is the electron charge and S is the spin |⃗| ~ angular momentum of the electron which Dirac showed obeyed S = 2 . The predicts g = 2 but the experimental value is very slightly larger (see discussion box below). Defining a dimensionless spin angular ⃗ ~ momentum vector ⃗σ via S = 2⃗σ, we have g ⃗µ = − µ ⃗σ, (1.7) 2 B where the Bohr magneton is defined by µ = e~ . The energy of a bar B 2me magnet in a magnetic field B⃗ depends on the orientation of the bar magnet CHAPTER 1. INTRODUCTION 9

Figure 1.2: Plaque memorializing the 1922 Stern-Gerlach experiment. From Bretislaw Friedrich and Dudley Herschbach, Physics Today, December, 2003. relative to the field g U = −µ · B⃗ = µ B⃗ · ⃗σ. 2 B The magnetic field supplies a torque that tries to align the magnetic moment with the B⃗ field. Because of the negative charge of the electron, its angular momentum wants to anti-align with the magnetic field.

Box 1.2. Quantum fluctuation corrections to the electron magnetic moment Dirac’s equation yields g = 2 exactly, but experimentally g differs slightly from that value due to subtle quantum effects called ‘vacuum fluc- tuations,’ in which the electron emits and quickly reabsorbs photons. The current experimental estimate [Hanneke et al., Physical Review A 83, 052122 g ≈ (2011)] for the value is 2 1.00115965218073 with an uncertainty of about 0.28 parts per trillion. The fact that g −2 can be measured so accurately and it agrees with theoretical predictions of quantum electrodynamics to many significant digits is one of the great triumphs of the quantum theory.

The spin angular momentum of the electron makes it behave like a gyro- scope. Recall that gyroscopes respond to torques by precssing. In the case of an electron in a magnetic field, the angular momentum vector of the electron precesses around the direction defined by the magnetic field. Thus if the magnetic field points in thez ˆ direction, Sz remains constant, and Sx and Sy oscillate sinusoidally (and 90 degrees out of phase with each other) at CHAPTER 1. INTRODUCTION 10

µB frequency ωB = g ~ B. For a field of 1 Tesla, this corresponds to an angular 9 frequency ωB ≈ (2π) 28.025 × 10 Hz.

Box 1.3. Precession of a gyroscope Consider the gyroscope shown in Fig. 1.3 consisting of a disk of mass M with angular momentum L⃗ spinning on an axle. The axle is supported at the origin and the force of gravity F⃗ = −Mgzˆ acts on the center of mass of the disk located at point R⃗, producing a torque ⃗τ = R⃗ × F⃗ = RF sin θφˆ in the azimuthal directionφ ˆ. Recognizing that the gravitational potential energy has the form U(θ) = Mgr cos θ, we − dU can write the torque in the form ⃗τ = dθ φˆ. We see that this torque is directed perpendicular to the angular momentum and so causes the angular momentum vector to precess in time (i.e. to rotate around thez ˆ axis). For an electron spin in a magnetic field, the magnetostatic energy is

g e ⃗ ⃗ g U(θ) = B · S = µBB cos θ 2 me 2

g so the torque is ⃗τ = 2 µBB sin θφ.ˆ The spin angular momentum precesses dS⃗ × ⃗ around thez ˆ axis according to dt = ωBzˆ S. Note that, unlike a macroscopic gyroscope, the electron spin does not undergo nutation. Can you think why this might be? It is related to the fact that the moment of inertia is such that the angular momentum vector of the gyroscope is not parallel to its axle (i.e., not solely due to the spin) if θ˙ ≠ 0. For a point particle like the electron, this cannot occur.

This rapid precession is an interesting phenomenon in its own right, but the details are not important for the Stern-Gerlach experiment. What counts is that because of the precession, the component of the spin parallel to the magnetic field is a constant of the motion, and so the energy is µBB cos(θ). Recall from basic chemistry that the allowed orbitals for electrons in an atom are generally doubled occupied by pairs of electrons with opposite spin. This is true for silver which has 46 paired electrons and one unpaired electron in the 5s orbital. (s means the orbital angular momentum is zero so we don’t have to worry about orbital angular momentum affecting the magnetic moment and are free to concentrate on the spin.) There is no reason to believe that the spin angular momentum of the silver atoms exiting the oven in the Stern-Gerlach experiment should have any preferred orientation foor their magnetic moment, it is reasonable to assume that cos θ is randomly and uniformaly distributed between -1 and CHAPTER 1. INTRODUCTION 11

G L

G R

G F Mgzˆ

Figure 1.3: Gyroscope precessing under a torque produced by gravity. The torque is into the page and the precession of the angular momentum vector occurs as a slow rotation dL⃗ × ⃗ − ⃗ × ⃗ ⃗ at rate Ωˆz that causes dt = Ωˆz L = τ = MgR zˆ. Using R = L(R/L) yields Ω = MgR/L. Notice that the precession frequency rate is independnent of the polar angle θ between R⃗ and the vertical directionz ˆ. The same is true for electron spins in a magnetic field.

+1. Thus the energy of the atoms in the magnetic field should be uniformaly ± g distributed between 2 µBB. Very surprisingly however, as shown in Fig. 1.4 one finds that the energy (known as the Zeeman energy) only takes on the two extremal values and never any value in between. Stern and Gerlach obtained this surprising result by sending the silver atoms through a non-uniform magnetic field whose magnitude had a strong gradient in thez ˆ direction3

∇|⃗ B⃗ | = λz.ˆ (1.8)

This caused the Zeeman energy to vary with height and produced a spin- dependent force g g F⃗ = −∇⃗ U = − µ ∇⃗ (|B⃗ | cos θ) = − µ λ cos θz,ˆ (1.9) 2 B 2 B that deflected the beam maximally upward or downward as if the only allowed values of cos θ were ±1. This surprising result indicated that if you ask the question, “Is the spin pointing up or down on the Bloch sphere?,” the answer is always ‘Yes, the z component of the (dimensionless) spin angular momentum is either σz = +1

3Recall from Maxwell’s equations that ∇ · B⃗ = 0. This constraint forces the magnetic field to have a form like B⃗ = B0(0, −λy, 1 + λz). This is a minor complication that does not significantly affect the results of the experiment for atoms lying close to the beam axis (x, 0, 0). CHAPTER 1. INTRODUCTION 12

E g  P B 2 B

B

g  P B 2 B

Figure 1.4: Zeeman energy of an electron in a magnetic field. Classically the energy can g − take on any value in the interval 2 µBB[ 1, 1], but quantum experiments show that only ± g the extremal values 2 µBB are ever observed. or σz = −1 and never anything else.’ For simplicity, let us refer to this version of the Stern-Gerlach experiment as ‘asking the Z question.’ By rotating the Stern-Gerlach magnet one could put the field gradient in another direction, say X. Now one finds that the component of the angular momentum in the direction of the field gradient is σx = ±1 as if the spin can only be parallel or antiparallel to thex ˆ direction! It seems that what state you find, depends on what you decide to measure. Once again, ‘we get what we see, not see what we get.’ To illustrate just how peculiar this is, consider the setup shown in Fig. 1.5. Because the oven presumably evaporates silver atoms with no preferred spin orientation, the first Z measurement randomly produces σz = ±1 with equal probabilities. However what happens next depends on what you measure! It is impossible for all three components of the spin vector to have definite values because they are incompatible . If you measure σx (say) you always get ±1 and you have no idea what the state was before you measured it. Afterwards you know the state has collapsed onto a unique state compatible with your measurement result, because if you measure the same component again you always get the same result. All of these peculiar results can be explained by the ansatz in Eq. (1.4) that the state with spin angular momentum pointing in the directionn ˆ = (sin θ cos φ, sin θ sin φ, cos θ) on the Bloch sphere is a superposition of the up CHAPTER 1. INTRODUCTION 13

a) 100% n 50% Z p n 0% Z p 50%

b) 50% o 50% X m n Z p 50%

Figure 1.5: A silver atom evaporated in the oven flies into a Stern-Gerlach measurement apparatus. A Z measurement randomly produces σz = ±1 with equal probability. (a) If we select those atoms with σz = +1, a second Z measurement yields σz = +1 100% of the time. (b) If instead we follow the Z measurement with an X measurement, we randomly obtain σx = ±1 with equal 50% probabilities. If we were to add second measurement of σx it would agree with the first. However if followed the X measurement with another Z measurement, the result would be completely random even though we had selected σz = +1 atoms in the first measurement. This illustrates the fact that Z and X measurements are fundamentally incompatible. and down states θ θ |ψ⟩ = |nˆ⟩ = cos | ↑⟩ + eiφ sin | ↓⟩. (1.10) 2 2 To see why this is so, consider the state with spin pointing in the +ˆx direction (corresponding to θ = π/2, φ = 0) 1 | + X⟩ = | →⟩ = √ [| ↑⟩ + | ↓⟩]. (1.11) 2

This is a coherent√ superposition of up and down with equal probability am- plitudes 1/ 2. Hence if we were to measure( ) Z we would randomly collapse 2 1 1 the state to | ↑⟩ with probability p↑ = √ = and to state | ↓⟩ with prob- ( ) 2 2 2 1 1 z ability p↓ = √ = . If the measurement result happens to be σ = +1, 2 2 then the state collapses to | ↑⟩, and a second Z measurement would also yield |σz in agreement with the first. CHAPTER 1. INTRODUCTION 14

Similarly, we could consider the state with spin pointing in the −xˆ direc- tion (corresponding to θ = π/2, φ = π) 1 | − X⟩ = | ←⟩ = √ [| ↑⟩ − | ↓⟩]. (1.12) 2 A Z measurement on this state also randomly yields σz = ±1 with equal probabilities. Suppose that the first Z measurement yields the value σz = +1 as shown in Fig. 1.5b. Then the state has collapsed to | ↑⟩. How do we predict what will happen if we were to make an X measurement? Using Eqs. (1.11-1.12) we see that we can write the state | ↑⟩ as the following superposition 1 | ↑⟩ = √ [| →⟩ + | ←⟩]. (1.13) 2 Thus we see that if we start with the state | ↑⟩ and make an X measurement we will randomly obtain σx = ±1 with equal probability just as illustrated in Fig. 1.5b. Similarly we can write 1 | ↓⟩ = √ [| →⟩ − | ←⟩] (1.14) 2 and again find that an X measurement will give a random result σx = ±1 with equal probabilities. For completeness we also note that states with spin aligned along the ±yˆ direction are given by Eq. (1.10) with θ = π/2, φ = ±π/2 1 | + Y ⟩ = √ [| ↑⟩ + i| ↓⟩] (1.15) 2 1 | − Y ⟩ = √ [| ↑⟩ − i| ↓⟩]. (1.16) 2 We can express the Z states in terms of these as 1 | ↑⟩ = √ [| + Y ⟩ + | − Y ⟩] (1.17) 2 −i | ↓⟩ = √ [| + Y ⟩ − | − Y ⟩]. (1.18) 2 This tells us that, just as Z and X measurements are incompatible, so are Z and Y measurements. CHAPTER 1. INTRODUCTION 15

There is a general method for expressing quantum states in terms of different orthogonal basis states which we will learn about in the next section. This method makes it very easy to derive expressions like those in Eqs. (1.17- 1.18).

Box 1.4. Predicting Measurement Outcomes In order to correctly pre- dict the probability of different measurement outcomes, you must express the given state |ψ⟩ in the basis appropriate to the measurement you wish to make. Thus to find the probability that a measurement of σy yields +1 or −1, you must express |ψ⟩ in the basis of | ± Y ⟩ and then apply the to find the probabilities from the probability amplitudes in the Y basis. We will shortly learn more formally how to do this.

Exercise 1.1. Express the states | ± Y ⟩ in terms of | ± X⟩. If you mea- sure σy = +1, what is the probability that a subsequent X measurement will yield σx = −1? Are X and Y measurements compatible? Chapter 2

Introduction to Hilbert Space

We have seen that the state of an electron whose spin points in a specified direction can be written as a linear combination of a pair of ‘basis states.’ These basis states could for example be | ↑⟩ and | ↓⟩

|ψ⟩ = α| ↑⟩ + β| ↓⟩, (2.1) where α and β are complex coefficients. The mathematical space of allowed states for an electron spin is a vector space over the field of complex num- bers and is referred to as the system’s Hilbert space. We are used to writing ordinary vectors as ‘row vectors’ (x, y) or (x, y, z). It is traditional however in quantum mechanics to represent vectors as ‘column vectors.’ Thus for ex- ample the basis states corresponding to Z measurements can be represented as ( ) 1 | ↑⟩ = (2.2) 0 ( ) 0 | ↓⟩ = (2.3) 1 which gives ( ) α |ψ⟩ = . (2.4) β

It will be useful to define an ‘inner product’ between pairs of vectors in Hilbert space that is analogous to the ordinary dot product. The inner

16 CHAPTER 2. INTRODUCTION TO HILBERT SPACE 17 product between ( ) α′ |ψ′⟩ = α′| ↑⟩ + β′| ↓⟩ = (2.5) β′ is defined to be (using the Dirac notation to be explained further shortly)

∗ ∗ ⟨ψ′|ψ⟩ = α′ α + β′ β. (2.6)

For ordinary vectors A⃗ ·B⃗ = B⃗ ·A⃗, but notice that for a complex vector space we have to be careful about the order since

⟨ψ′|ψ⟩ = (⟨ψ|ψ′⟩)∗. (2.7)

The complex conjugation means that the inner product of any vector with itself is real

⟨ψ|ψ⟩ = |α|2 + |β|2 = 1, (2.8) where the second equality follows from the Born interpretation of the magni- tude squared of probability amplitudes as measurment outcome probabilities. Dirac referred to his notation for state vectors as the ‘bracket’ notation. A quantum states is represented by ‘ket’ |ψ⟩. Associated with this vector is a dual ‘bra’ vector defined to be the following row vector

( )† α ( ) ⟨ψ| = = α∗ β∗ , (2.9) β where † indicates the complex conjugate of the transpose. Thus ( ) ( ) α∗ β∗ α′ ⟨ψ|ψ′⟩ = = α∗α′ + β∗β′, (2.10) β′ where the last equality follows from the ordinary rules of matrix multiplica- tion (here applied to non-square matrices). Even though the inner product can be complex, we can still think of it as telling us something about the angle between two vectors in Hilbert space. Thus for example

⟨↑ | ↓⟩ = ⟨+X| − X⟩ = 0. (2.11) CHAPTER 2. INTRODUCTION TO HILBERT SPACE 18

These pairs of vectors are orthogonal even though their corresponding spin vectors are co-linear (anti-parallel). Thus opposite points on the Bloch sphere correspond to orthogonal vectors in Hilbert space. [This is closely tied to the half angles that appear in Eq. (1.10).] If two state vectors are orthogonal then the states are completely distinguishable. A system prepared in one of the states can be measured to uniquely determine which of the two states the system is in. Of course if the system is in a superposition of two orthogonal states, the measurement result can be random. Conversely, if a system is in one of two states that are not orthogonal, it is not possible to reliably determine by measurement which state the system is in.

2.1 Linear Operators on Hilbert Space

An defines a mapping between the Hilbert space back into the same Hilbert space. In quantum mechanics we will deal with linear operators, which for the case of a single spin are simply 2x2 matrices that when mul- tiplied into a column vector representing a state, produce another column vector representing another state. It turns out that physical observables in quantum mechanics are represented by Hermitian matrices. We cannot use numbers to represent physical observables because observables do not have values before we measure them. Instead observables have the potential to have different values upon measurement. It turns out that matrices can be used to represent this peculiar situation. This takes some getting used to, so we will proceed with some examples. It turns out that the z component of the spin angular momentum is represented in the following way ( ) 1 0 σz = . (2.12) 0 −1 CHAPTER 2. INTRODUCTION TO HILBERT SPACE 19

Now notice the following interesting properties of this matrix ( )( ) 1 0 1 σz| ↑⟩ = 0 −1 0 ( ) 1 = = (+1)| ↑⟩ (2.13) 0 ( )( ) 1 0 0 σz| ↓⟩ = 0 −1 1 ( ) 0 = = (−1)| ↓⟩. (2.14) −1

These equations tell us that | ↑⟩ is an eigenstate of σz with eigenvalue +1 and that | ↓⟩ is an eigenstate of σz with eigenvalue -1. The eigenvectors of an operator are those vectors that are mapped into themselves under the operation, up to a constant factor called the eigenvalue. It turns out that the possible measurement results for some physical ob- servable are given by the eigenvalues of the operator corresponding to that observable. Hence, we see that for the case of σz the possible measurement results must be ±1 in agreement with experiment. Furthermore, it turns out that if a given quantum state is an eigenstate of a particular operator with eigenvalue λ then the measured value of the observable will not be random, but in fact will always be exactly λ. Let us consider what happens when a state is not an eigenstate of an observable. Consider for example

|ψ⟩ = α| ↓⟩ + β| ↑⟩ (2.15) σz|ψ⟩ = −α| ↓⟩ + β| ↑⟩. (2.16)

If α and β are both non-zero, this is clearly not an eigenvector. But this is consistent with the fact that the measurement result will be random with 2 2 probability p↑ = |β| of being +1 and probability p↓ = |α| of being -1. CHAPTER 2. INTRODUCTION TO HILBERT SPACE 20

Box 2.1. Eigenvalues, Eigenvectors and Measurements It is important to note that the act of measurement of an observable is not simply repre- sented by multiplying the quantum state |ψ⟩ by the operator corresponding to the observable being measured (e.g. σz). In particular, σz|ψ⟩ is not the state of the system after the measurement of σz. The state always randomly collapses to one of the eigenstates of the operator being measured. Further- more, the measurement result is always one of the eigenvalues of the operator being measured and the particular state to which the system collapses is the eigenvector corresponding to the eigenvalue that is the measurement result. Thus if the result of the measurement of σz is +1, the state collapses to | ↑⟩. If the result of the measurement is −1, the state collapses to | ↓⟩. From the Born rule, the probabilities that the state collapses to | ↑⟩ and | ↓⟩ are respectively

p↑ = ⟨ψ| ↑⟩⟨↑ |ψ⟩ (2.17)

p↓ = ⟨ψ| ↓⟩⟨↓ |ψ⟩. (2.18)

How do we know that measurement collapses the state onto one of the eigen- vectors of the operator being measured? This follows from the experimental fact that a Stern-Gerlach Z measurement (say) that gives a certain result, will give exactly the same result if we make a second Z measurement. The most general possible state is a superposition of the two eigenstates of σz of the form |ψ⟩ = α| ↑⟩ + β| ↓⟩. In order for the second Z measurement to not be random, we must have (from the Born rule) that |α|2 and |β|2 cannot both be non-zero. We have to be in one eigenstate or the other so that the probability of getting the same measurement result is 100%. Thus the first Z measurement has to collapse the state onto one of the eigenstates of Z.

Recall that measurement collapses the state, so if we want to know the average measurement result for the state we have to prepare many copies and measure each copy only once. The average measurement result will of 2 2 course be (+1)p↑ + (−1)p↓ = |β| − |α| . Let us compare this quantity to the so-called ‘expectation value’ of the operator σz in the state |ψ⟩ which is defined by the expression ⟨ψ|σz|ψ⟩. What we mean by this expression is compute the state σz|ψ⟩ and then takes its inner product (‘overlap’) with CHAPTER 2. INTRODUCTION TO HILBERT SPACE 21

|ψ⟩: ⟨ψ|σz|ψ⟩ = (α∗⟨↑ | + β∗⟨↓ |)(+α| ↑⟩ − β| ↓⟩ = +|α|2⟨↑ | ↑⟩ − α∗β⟨↑ | ↓⟩ + β∗α⟨↓ | ↑⟩ − |β|2⟨↓ | ↓⟩ = |α|2 − |β|2. (2.19) Or equivalently in matrix notation ( )( ) ( ) z ∗ ∗ 1 0 ψ↑ ⟨ψ|σ |ψ⟩ = ψ↑ ψ↓ (2.20) 0 1 ψ↓ 2 2 2 2 = |ψ↑| − |ψ↓| = |α| − |β| . (2.21) Thus we have the nice result that the average measurement result for some observable is simply the expectation value of the observable (operator) in the quantum state being measured. Can we now say something about how random the measurement result will be? Let us begin by reminding ourselves about some basic facts about classical random variables. Suppose some random variable ξ takes on values from the set {v1, v2, . . . , vM } and value vj occurs with probability pj. Then the mean value is given by ∑M ¯ ξ = pjvj. (2.22) j=1 where the overbar indicates statistical average. One measure of how random ξ is the variance defined by Var(ξ) ≡ (ξ − ξ¯)2 = ξ2 − ξ¯2 (2.23) [ ] ∑M ∑M 2 2 − = pjvj pjvj . (2.24) j=1 j=1 We see that the variance is the mean square deviation from the average and so is a measure of the size of the random fluctuations in the ξ. Exercise 2.1. Statistical Variance

a) Derive Eq. (2.24).

b) Assuming that all the eigenvalues vj are distinct (i.e. non- degenerate), prove that the variance vanishes if and only if one of the pj’s is unity and the rest vanish (so that ξ is not random). If one of the eigenvalues is m-fold degenerate, then the correspond- ing m probabilities must add up to unity and the rest must vanish. CHAPTER 2. INTRODUCTION TO HILBERT SPACE 22

Let us now turn to the quantum case. Let observable Q have eigenvalues {v1, v2, . . . , vM }. (So far we have only considered a single spin for which M = 2. In more general cases (e.g. multiple spins), the Hilbert space M can be larger.) The analog of Eq. (2.24) for the variance of the measurement results for Q in state |ψ⟩ is

Var(Q) = ⟨ψ|Q2|ψ⟩ − ⟨ψ|Q|ψ⟩2. (2.25)

Now notice that if |ψ⟩ is an eigenvector of Q (say the mth eigenvector)

Q|ψ⟩ = vm|ψ⟩, (2.26) then

2 ⟨ | ⟩ − ⟨ | ⟩ 2 Var(Q) = vm ψ ψ [vm ψ ψ ] = 0. (2.27)

Thus only superpositions of states (in the basis appropriate to the measure- ment!) with different eigenvalues will have randomness in the measurement results. This is entirely consistent with the experimental facts described above for a single spin. Exercise 2.2. Since the eigenvalues of σz are ±1, we expect that (σz)2 has both of its eigenvalues equal to unity. From the matrix form of σz prove that its square is the identity matrix ( ) 1 0 (σz)2 = Iˆ = . 0 1 CHAPTER 2. INTRODUCTION TO HILBERT SPACE 23

Box 2.2. Observables as Hermitian operators Hermitian matrices (that is, matrices obeying M † = M, where again † indicates transpose followed by complex conjugation) are guaranteed to have real eigenvalues and that their eigenvectors for a complete basis set that spans the Hilbert space (more on what this means later). Since the measured values of physical observables are always real, physical observables are always represented by Hermitian matrices. Let us now demonstrate that Hermitian matrices have real eigenvalues and that their eigenvectors are orthogonal. Consider the jth and kth (normalized) eigenvectors

H|ψj⟩ = λj|ψj⟩, (2.28)

H|ψk⟩ = λk|ψj⟩. (2.29) Taking the adjoint of the second equation we obtain ⟨ | † ∗⟨ | ψk H = λk ψk . (2.30) ∗ NOTE THE ERROR IN SUSSKIND. HE WRITES λk INSTEAD OF λk. Assuming H† = H, can now use Eq. (2.28) and Eq. (2.30) to obtain

⟨ψk|H|ψj⟩ = λj⟨ψk|ψj⟩, (2.31) ⟨ | | ⟩ ∗⟨ | ⟩ ψk H ψj = λk ψk ψj . (2.32) Subtracting these yields

− ∗ ⟨ | ⟩ 0 = (λj λk) ψk ψj . (2.33)

For the case j = k, we know that (by construction) ⟨ψj|ψj⟩ = 1, and hence the imaginary part of λj must vanish. Thus all the eigenvalues of an Hermitian operator are real. If for j ≠ k, the eigenvalues are non-degenerate (i.e., λj ≠ λk), then Eq. (2.33) requires ⟨ψk|ψj⟩ = 0 and so the two eigenvectors must be orthogonal. Thus if the full spectrum is non-degenerate, the set of eigenvectors is orthonormal: ⟨ψk|ψj⟩ = δkj. If M eigenvalues are degenerate, then any superposition of the M eigenvec- tors is also an eigenvector. If they are not orthogonal, they can be made orthogonal by taking appropriate linear combinations of the set of M eigen- vectors (via the so-called Gram-Schmidt procedure). CHAPTER 2. INTRODUCTION TO HILBERT SPACE 24 2.2 Dirac Notation for Operators

We have seen that in Dirac’s bracket notation, the inner product ⟨Φ|Ψ⟩ is a (possibly complex) number. The outer product G = |Ψ⟩⟨Φ| turns out to be an operator on the space. To see why, simply operate with G on an arbitrary state |χ⟩ G|χ⟩ = |Ψ⟩⟨Φ|χ⟩ = g|Ψ⟩, (2.34) where g is the complex number representing the inner product of |Φ⟩ and |Ψ⟩, g = ⟨Φ|χ⟩. (2.35) Hence when applied to any vector in the Hilbert space, G returns another vector in the Hilbert space. Thus it is an operator and indeed it is a linear operator since G(α|χ⟩ + β|ϕ⟩) = αG|χ⟩ + βG|ϕ⟩. (2.36) As a specific example, consider the operator G = | ↑⟩⟨↑ | − | ↓⟩⟨↓ |. (2.37) Clearly G| ↑⟩ = (+1)| ↑⟩ (2.38) G| ↓⟩ = (−1)| ↓⟩, (2.39) so it must be that G = σz. To confirm this let us write G out in matrix form ( ) ( ) ( ) ( ) 1 1 0 0 0 1 G = − (2.40) 0 1 ( ) 1 0 = , (2.41) 0 −1 where the last result follows from the rules of matrix multiplication and agrees with Eq. (2.12). The above result is a particular case of the general fact that in Dirac’s notation, Hermitian operators take on a very simple form

∑M V = vj|ψj⟩⟨ψj|, (2.42) j=1 CHAPTER 2. INTRODUCTION TO HILBERT SPACE 25

where |ψj⟩ is the jth eigenvector of V with eigenvalue vj. Exercise 2.3. Show that the representation of the operator V in Eq. (2.42) correctly reproduces the required property of V that

V |ψm⟩ = vm|ψm⟩,

for every eigenvector |ψm⟩ of V .

Exercise 2.4. The three components of the (dimensionless) spin an- gular momentum vector ⃗σ = (σx, σy, σz) are each physical observables. The 2 × 2 matrices representing these operators are known as the . We have already seen the matrix representation for σz in the up/down (Z) basis ( ) +1 0 σz = . (2.43) 0 −1

Using the known form of the states |±X⟩ from Eq. (1.11) and Eq. (1.12) and | ± Y ⟩ in Eqs. (1.15-1.16), show that in the Z basis ( ) 0 +1 σx = (2.44) +1 0 ( ) 0 −i σy = . (2.45) +i 0

2.3 Orthonormal bases for electron spin states

It is possible to show that the quantum states |±nˆ⟩ corresponding to any pair of oppositely directed unit vectors ±nˆ on the Bloch sphere are orthogonal ⟨−nˆ| +n ˆ⟩ = 0. (2.46) Since were are dealing with a Hilbert space of only two and we have an orthonormal pair of states in the Hilbert space, they constitute a complete basis for expressing any vector in the Hilbert space. It is straightforward to derive the completeness relation | +n ˆ⟩⟨+ˆn| + | − nˆ⟩⟨−nˆ| = I,ˆ (2.47) CHAPTER 2. INTRODUCTION TO HILBERT SPACE 26 where Iˆ is the identity operator whose matrix representation is ( ) 1 0 Iˆ = . (2.48) 0 1

This is a powerful result because it allows us to express any state |ψ⟩ in any basis by simply writing

|ψ⟩ = Iˆ|ψ⟩ = | +n ˆ⟩⟨+ˆn|ψ⟩ + | − nˆ⟩⟨−nˆ|ψ⟩. (2.49)

We see immediately the coefficients in the expansion of the state |ψ⟩ in the basis are simply computed as the inner products of the basis vectors with the state |ψ⟩. The results above are very reminiscent of how we find the representation of an ordinary vector, say a position vector in 2D. We may have some orthogonal basis vectors for our coordinate system, for examplex, ˆ yˆ, or a rotated set ˆi, ˆj. We can represent any vector as

⃗r = (rx, ry) =x ˆ(ˆx · ⃗r) +y ˆ(ˆy · ⃗r) (2.50) = ˆi(ˆi · ⃗r) + ˆj(ˆj · ⃗r). (2.51) This suggests we can think of the identity transformation as

Iˆ =x ˆxˆ +y ˆyˆ = ˆiˆi + ˆjˆj, (2.52) where we interpretx ˆxˆ applied to a vector Vˆ to meanx ˆ(ˆx · V⃗ ). Note the similarity between Eq. (2.52) and Eq. (2.47). It is useful at this point to discuss the concepts of projections and projec- tors. Recall that when we project an ordinary vector ⃗r onto the x axis, we simply remove all components of ⃗r other than the x component. Thus the ⃗ projection of ⃗r onto the x axis is the vectorxV ˆ x =x ˆ(ˆx · V ). We can think of Px =x ˆxˆ as a projection operator (or projector) because ⃗ ⃗ PxV = (ˆxxˆ)V =x ˆ(ˆx · V ). (2.53)

Notice that Px satisfies the defining characteristic of projection operators

2 (Px) = Px. (2.54)

This has a simple interpretation–once the vector is projected onto the x axis, further projection doesn’t do anything. CHAPTER 2. INTRODUCTION TO HILBERT SPACE 27

We can think of the shadow of an object cast onto the ground as the pro- jection of the objection onto the horizontal (xy) plane. This is accomplished by the projection operator ˆ Pxy =x ˆxˆ +y ˆyˆ = I − zˆz,ˆ (2.55) where Iˆ =x ˆxˆ +y ˆyˆ +x ˆzˆ is the identity for the 3D case. This shows us that projection onto the xy plane simply removes the component of the vector normal to the plane. Despite this more complicated form, it is straightfor- 2 ward to show that (Pxy) = Pxy as required. The key to this is that the basis vectorsx, ˆ y,ˆ zˆ are orthonormal. The analogous projector onto the vector |nˆ⟩ in the Hilbert space describ- ing the spin of an electron is simply

Pnˆ = |nˆ⟩⟨nˆ|. (2.56)

It is straightforward to show that this is idempotent PnˆPnˆ = Pnˆ as required. Exercise 2.5. Derive Eq. (2.46).

Exercise 2.6. Derive Eq. (2.47). Eq. (2.47) along with the Born rule tells us that means that, given an arbitrary state ψ⟩, a measurement asking the question ‘Is this the state |nˆ⟩ 2 will be answered ‘yes’ with probabilitygiven ⟨ψ|Pnˆ|ψ⟩ = |⟨nˆ|ψ⟩| . Exercise 2.7. Show that | ± nˆ⟩ is an eigenvector of

H =n ˆ · σ = (ˆn · xˆ)σx + (ˆn · yˆ)σy + (ˆn · zˆ)σz

with eigenvalue ±1

H| +n ˆ⟩ = (+1)| +n ˆ⟩ (2.57) H| − nˆ⟩ = (−1)| − nˆ⟩ (2.58)

This is simply a reflection of the rotation invariance of space. We can align our Stern-Gerlach magnet in the arbitrary directionn ˆ and the measured state of the spin will be either | +n ˆ⟩ or | − nˆ⟩. CHAPTER 2. INTRODUCTION TO HILBERT SPACE 28

2.3.1 Gauge Invariance We have seen that any two states at opposite points on the Bloch sphere form an orthonormal basis that can be used to represent any state in the 2D Hilbert space of a single spin. One confusing aspect of all this that we have not yet discussed, is the following. The states |±nˆ⟩ in the Hilbert space corresponding to the Bloch sphere unit vectors ±nˆ are not unique. Each basis iξ+ vector can be multiplied by a different arbitrary phase factor |v+⟩ = e |+ˆn⟩ iξ− and |v−⟩ = e | − nˆ⟩ and we would still have a perfectly good orthonormal basis obeying

⟨v+|v+⟩ = ⟨v−|v−⟩ = 1, (2.59)

⟨v−|v+⟩ = 0, (2.60) and the completeness relation ˆ I = |v+⟩⟨v+| + |v−⟩⟨v−|. (2.61)

The particular choice of phase factors cancels out in the above expressions. The specific representation of one other operator (besides the identity) is also independent of the ‘gauge’ choice that we make by picking particluar phase factors, namely the (diagonal) operator that is measured by a Stern-Gerlach magnet oriented in then ˆ direction

nˆ · σ = |v+⟩⟨v+| − |v−⟩⟨v−| (2.62) = | +n ˆ⟩⟨+ˆn| − | − nˆ⟩⟨−nˆ|, (2.63) is invariant and the probability of the two measurement results is unaffected by the gauge choice. However non-diagonal operators such as the spin flip operator ˆ Q = |v+⟩⟨v−| + |v−⟩⟨v+| (2.64) − − − = e+i(ξ+ ξ−)| +n ˆ⟩⟨−nˆ| + e i(ξ+ ξ−)| − nˆ⟩⟨+ˆn| (2.65) are gauge dependent when expressed in the | ± nˆ⟩ basis. Of course the state vectors also become gauge dependent

|ψ⟩ = α|v+⟩ + β|v−⟩ (2.66) = αeiξ+ | +n ˆ⟩ + βeiξ− | − nˆ⟩, (2.67) and so nothing changes in the physics. CHAPTER 2. INTRODUCTION TO HILBERT SPACE 29

Interestingly, even the apparently fixed definition for | +n ˆ⟩ in Eq. (1.10) is ambiguous when it comes to writing down the other basis vector | − nˆ⟩. Ifn ˆ = (sin θ cos φ, sin θ sin φ, cos θ), then we can arrive at the oppositely directed unit vector −nˆ via two different routes: i) θ → θ + π, φ → φ, or ii) θ → π − θ, φ → φ + π. Consider for example the case θ = π/2, φ = 0 which gives 1 |nˆ⟩ = | + X⟩ = √ [| ↑⟩ + | ↓⟩] . (2.68) 2 Both transformation (i) and (ii) taken ˆ to −nˆ on the Bloch sphere but the two resulting states in the Hilbert space differ by a gauge choice (i.e., a phase) 1 | − nˆ⟩ = | − X⟩ = −√ [| ↑⟩ − | ↓⟩] Method(i) (2.69) 2 1 = +√ [| ↑⟩ − | ↓⟩] Method(ii) (2.70) 2 Method (ii) self-consistently keeps θ in the range 0 ≤ θ ≤ π and is the standard gauge choice. Going back to Eq. (1.12) and Eq. (1.16), we see that Method (ii) was used in to define | − X⟩ and | − Y ⟩ and yields the standard form of the Pauli matrices displayed in Ex. 2.4.

2.4 Rotations in Hilbert Space

In order to build a quantum computer we need to have complete control over the quantum states of a system of many qubits. For the moment however let us consider how we might create an arbitrary state of a single qubit starting from some fiducial state, typically taken to be either | ↑⟩ or | ↓⟩ in the standard basis. Since the most general state is a superposition state |nˆ⟩ corresponding to an arbitrary pointn ˆ on the Bloch sphere, we need to be able to perform rotations in Hilbert space that correspond to the rotation of the Bloch vector some initial position to any desired position on the Bloch sphere. Being able to prepare arbitrary states is a requirement for carrying out a quantum computation. Because every state in the single- or multi-qubit Hilbert space has the same length ⟨ψ|ψ⟩ = 1, rotations are the only operations we need to move throughout the entire Hilbert space. There is no concept of ‘translations’ in Hilbert space. If we add a vector |ϕ⟩ to the vector |ψ⟩ we of course obtain CHAPTER 2. INTRODUCTION TO HILBERT SPACE 30

G v1 vG G 3 v2 vG 2 GG G v1 v 2 v1

Figure 2.1: Two unit vectors ⃗v1,⃗v2 whose end points lie on the unit circle in 2D. The sum of the vectors ⃗v1 + ⃗v2 (generically) has length squared W = 2 + 2(⃗v1 · ⃗v2) ≠ 1. The normalized vector ⃗v = √1 (⃗v + ⃗v ) does lie on the unit circle and can be obtained by a 3 W 1 2 rotation applied to either ⃗v1 or ⃗v2. another element of the complex vector space, but generically |ψ⟩ + |ϕ⟩ is not properly normalized to unity. One can show that the normalized version of the state 1 |Λ⟩ = √ (|ψ⟩ + |ϕ⟩) , (2.71) W with

W = 2 + ⟨ψ|ϕ⟩ + ⟨ϕ|ψ⟩, (2.72) can be obtained from the initial state |ψ⟩ via a rotation. The idea is illus- trated schematically for ordinary 2D vectors in Fig. (2.1). It turns out that rotations in Hilbert space are executed via unitary op- erations. A unitary matrix U has the defining property

U †U = I, (2.73) where I is the identity matrix. Thus the inverse of U is simply the adjoint U −1 = U †. It turns out that unitary transformations preserve the inner products between vectors in Hilbert space, just as the more familiar orthog- onal rotation matrices preserve the angles (dot products) between ordinary vectors. To better understand rotations consider the representation of an arbitrary state vector in terms of some orthonormal basis {|j⟩; j = 1, 2, 3,...,N} span- ning the N-dimensional Hilbert space

∑N |ψ⟩ = ψj|j⟩. (2.74) j=0 CHAPTER 2. INTRODUCTION TO HILBERT SPACE 31

From the Born rule, the probability of measuring the system to be in the basis 2 state |j⟩ is given by pj = |ψj| . The requirement that the total probability be unity gives us the normalization requirement on the state vector ∑N 2 |ψj| = 1. (2.75) j=1 Now consider a linear operation U that preserves the length of every vector in the Hilbert space. ∑N ′ |ψ ⟩ = U|ψ⟩ = ψjU|j⟩, (2.76) j=1 ⟨ψ′|ψ′⟩ = ⟨ψ|U †U|ψ⟩ (2.77) ∑N ∗ ⟨ | † | ⟩ = ψkψj k U U j (2.78) j,k=1 ∑N ∑N | |2⟨ | † | ⟩ ∗ ⟨ | † | ⟩ = ψj j U U j + ψkψj k U U j . (2.79) j=1 j≠ k Since U preserves the length of every vector we have ⟨ψ|U †U|ψ⟩ = 1 (2.80) and ⟨j|U †U|j⟩ = 1 (2.81) for every basis vector |j⟩. Thus we must have

∑N ∗ ⟨ | † | ⟩ ψkψj k U U j = 0 (2.82) j≠ k for all possible choices of the set of amplitudes ψj; j = 1, 2, 3,...,N. Thus it must be that † ⟨k|U U|j⟩ = δkj, (2.83) where the Kronecker delta symbol δjk = 1 for j = k and 0 for j ≠ k. Equivalently, this means U †U = I. (2.84) CHAPTER 2. INTRODUCTION TO HILBERT SPACE 32

Thus the only linear operations that conserve probability for all states are unitary operations. It follows that unitary transformations preserve not only the inner product of states with themselves but also preserve the inner prod- ucts between any pair of states |ϕ′⟩ = U|ϕ⟩ (2.85) |ψ′⟩ = U|ψ⟩ (2.86) ⟨ϕ′|ψ′⟩ = ⟨ϕ|U †Uψ⟩ = ⟨ϕ|ψ⟩. (2.87) KEEP WORKING ON THIS. It turns out that in an ideal, dissipationless closed quantum system, the evolution of the system from its initial state at time 0 to its final state at time t is described by a unitary transformation. We can control the time evolution, and thus create different unitary operations, by applying control signals to our quantum system. The specifics of how this is done for different systems using beams, microwave pulses, magnetic fields, etc. will not concern us at the moment. We will for now simply postulate that the only operations available to us to control the quantum system are multiplication of the starting state by a unitary matrix to effect a rotation in Hilbert space. Being able to rotate states in Hilbert space can also be very useful for the purposes of measurement. If we literally had a Stern-Gerlach magnet that could be oriented in every possible direction, we could measure any component of the spin vector ⃗σ. In general however our qubit will not be (say) a silver atom whose magnetic moment we can measure with a Stern- Gerlach magnet. The two states of our qubit might be the ground and first excited electronic orbital states of an atom, or might be the ground and first excited state of a superconducting circuit. In these cases it often happens that the energy eigenstates of the system (i.e., of the Hamiltonian operator) constitute the only basis in which measurements can be made. Since we still have a two-level system, we have the same Hilbert space and the same operators, so the mathematics is the same, but the ‘spin’ should be thought of as a ‘pseudospin.’ The direction of the pseudospin vector on the Bloch sphere tells us about the superposition state of the qubit but has nothing to do with angular momentum or magnetic moment. Typically we represent the energy eigenstates in terms of the standard basis states | ↑⟩ and | ↓⟩. Thus the Hamiltonian (energy operator) is E + E E − E H = 0 1 Iˆ+ 1 0 σz, (2.88) 2 2 CHAPTER 2. INTRODUCTION TO HILBERT SPACE 33

which has eigenvalues E0 and E1. If we choose the zero of energy to be half way between the ground and excited state energies then E0 + E1 = 0 and we can drop the first term. If we are able to measure the energy (say) then the preferred measurement operator to which we have access is σz. If the qubit is in the state

|ψ⟩ = α| ↑⟩ + β| ↑⟩, (2.89) and we are able to prepare many copies of this state, a histogram of the measurement results Z = ±1 plus the Born rule allows us to deduce the values of |α|2 and |β|2. We cannot however deduce the relative complex phase of α and β. (Recall that WLOG we can take α to be real.) To fully determine the state we need (in general) to be able to measure all three components of ⃗σ. If we only have access to measurements of σz, then full state ‘tomography’ seems to be impossible. However if we prepend certain selected rotations of the state before making making the Z measurement we can achieve our goal. For example a rotation by π/2 around the y axis takes | + X⟩ to | − Z⟩ and | − X⟩ to | + Z⟩. Similarly a rotation by π/2 around the x axis takes | + Y ⟩ to | + Z⟩ and | − Y ⟩ to | − Z⟩. Thus we can measure all three components of the spin. In fact, we can rotate any state |nˆ⟩ into | + Z⟩ and thus measure the operatorn ˆ · ⃗σ. Before we learn how to rotate states in Hilbert space, let us review the familiar concept of rotations in ordinary space. For example if we start with an ordinary 3D unit vector on the Bloch sphere

nˆ = xxˆ + yyˆ + zzˆ = [sin θ cos φ xˆ + sin θ sin φ yˆ + cos θ zˆ] , (2.90) we can rotate it through an angle χ around the z axis to yield a new vector

nˆ ′ = [sin θ cos(φ + χ)x ˆ + sin θ sin(φ + χ)y ˆ + cos θ zˆ] , (2.91) which simply corresponds to a transformation of the polar coordinates θ → θ, φ → φ + χ. If we choose to representn ˆ as a column vector     x sin θ cos φ nˆ =  y  =  sin θ sin φ  , (2.92) z cos θ then, using the trigonometric identities cos(θ + χ) = cos(θ) cos(χ) − sin(θ) sin(χ) and sin(θ + χ) = sin(θ) cos(χ) + cos(θ) sin(χ), it is straight- CHAPTER 2. INTRODUCTION TO HILBERT SPACE 34 forward to show that the new vectorn ˆ ′ is represented by     x′ x ′  ′    nˆ = y = Rz(χ) y , (2.93) z′ z where Rz(χ) is a 3 × 3 ‘rotation matrix’   cos χ − sin χ 0   Rz(χ) = sin χ cos χ 0 . (2.94) 0 0 1 Of course, if we follow this by a rotation through angle −χ we must return ˆ the vector to its original orientation. Hence Rz(−χ)Rz(χ) = I or equivalently − −1 Rz( χ) = Rz (χ). From the fact that sin and cos are respectively odd and even in their arguments, it is straightforward to show that

−1 T Rz (χ) = Rz (χ). (2.95) Matrices whose transpose is equal to their inverse are called orthogonal and it turns out that rotations (for ordinary vectors) are always represented by orthogonal matrices. Two natural properties of rotations are that they preserve the length of vectors and they preserve the angle between vectors. These facts can be ′ ′ summarized in the single statement that if ⃗r1 = Rz(χ)⃗r1 and ⃗r2 = Rz(χ)⃗r2, ′ ′ then ⃗r1 · ⃗r2 = ⃗r1 · ⃗r2. That is, the dot product between any two vectors (including the case of the the dot product of a vector with itself) is invariant under rotations. The mathematical source of the preservation of lengths can be traced back to the fact that the determininant of an orthogonal matrix is unity. Let us now turn to rotations in Hilbert space. There, must be some connection to ordinary rotations, because rotation of the spin vector on the Bloch sphere is an ordinary (3D vector) rotation. However the Hilbert space is only two-dimensional, and unlike the example above where the standard basis vectors werex, ˆ yˆ andy ˆ, the standard basis vectors in the Hilbert space correspond to the orthogonal states | + Z⟩ = | ↑⟩ and | − Z⟩ = | ↓⟩. Fur- thermore, the inner product in Hilbert space involves complex conjugation unlike the case of the dot product for ordinary vectors. Hence we expect that rotation operations in Hilbert space will not look like 3D rotations of the unit vectors on the Bloch sphere. CHAPTER 2. INTRODUCTION TO HILBERT SPACE 35

Let us begin with rotations around the z axis. We know from the example above, that rotation of the 3D vectorn ˆ on the Bloch sphere by an angle χ around the z axis, simply corresponds to the transformation φ → φ + χ. From the standard quantum state representation in Eq. (1.10), we see that this simply changes the relative phase of the coefficients of | ↑⟩ and | ↓⟩. Let us therefore consider the following operator on the Hilbert space which does something very similar

−i χ σz Uz(χ) = e 2 . (2.96)

What do we mean by the exponential of a matrix? One way to interpret this expression is via a power series expansion ∞ [ ] ∑ 1 χ n U (χ) = −i [σz]n. (2.97) z n! 2 n=0

|χ| | z| The series should converge provided that 2 σ lies within the radius of convergence of the series expansion. Here |σz| means the absolute value of the largest eigenvalue of the matrix. Since |σz| = 1 and since the exponential is an entire function (it is analytic everywhere in the complex plane and hence the series has infinite radius of convergence), the series does converge1. Using the result of Ex. 2.2, we can rewrite Eq. (2.97) as [ ] [ ] ∑ 1 χ n ∑ 1 χ n U (χ) = −i Iˆ+ −i σz z n! 2 n! 2 n= even n= odd χ χ = cos Iˆ− i sin σz. (2.98) 2 2 Using the fact that σz| ↑⟩ = (+1)| ↑⟩ and σz| ↓⟩ = (−1)| ↓⟩, yields

−i χ Uz(χ)| ↑⟩ = e 2 | ↑⟩ (2.99) +i χ Uz(χ)| ↓⟩ = e 2 | ↑⟩. (2.100) Applying this to the standard state |nˆ⟩, and using the fact that σz| ↑⟩ =

1For infinite-dimensional matrices with unbounded eigenvalues, one has to be more careful with this analysis. CHAPTER 2. INTRODUCTION TO HILBERT SPACE 36

(+1)| ↑⟩ and σz| ↓⟩ = (−1)| ↓⟩, yields [ ] θ θ iφ Uz(χ)|nˆ⟩ = Uz(χ) cos | ↑⟩ + sin e | ↓⟩ (2.101) [ 2 2 ] θ −i χ θ i(φ+ χ ) = cos e 2 | ↑⟩ + sin e 2 | ↓⟩ (2.102) [2 2 ] −i χ θ θ i(φ+χ) = e 2 cos | ↑⟩ + sin e | ↓⟩ (2.103) 2 2 −i χ = e 2 |Rz(χ)ˆn⟩ (2.104) −i χ ′ = e 2 |nˆ ⟩. (2.105)

Thus the 2 × 2 complex matrix Uz(χ) correctly rotates the quantum state by an angle χ around the z axis of the Bloch sphere. Notice however that the resulting state differs from the standard state by an (irrelevant) global phase factor corresponding to a gauge change. − −1 † Notice that Uz( χ) = Uz (χ) = Uz (χ). Hence Uz(χ) is unitary, meaning that † ˆ Uz Uz = I. (2.106) All rotations in Hilbert space are unitary and it is straightforward to show that any unitary transformation U preserves the inner product in the Hilbert space. Consider two arbitrary states |Ψ⟩ and |Φ⟩ to which U is applied |Ψ′⟩ = U|Ψ⟩ (2.107) |Φ′⟩ = U|Φ⟩ (2.108) ⟨Φ′| = ⟨Φ|U † (2.109) ⟨Φ′|Ψ′⟩ = ⟨Φ|U †U|Ψ⟩ = ⟨Φ|Ψ⟩. (2.110) Thus unitary matrices for complex vector spaces preserve inner products hence are the analog of orthogonal matrices for real vector spaces. Exercise 2.8. Prove that the Paul matrices σx, σy, σz are both Hermi- tian and unitary.

Exercise 2.9. a) Prove that every operator that is both Hermitian and unitary squares to the identity. Using this, prove that if such operators are traceless, their spectrum contains only +1 and −1 and has equal numbers of each.

b) Conversely, prove the following lemma: Every operator that both squares to the identity and is Hermitian, is necessarily unitary. CHAPTER 2. INTRODUCTION TO HILBERT SPACE 37

Exercise 2.10. Every N × N unitary matrix can be written in the form

ˆ U = eiθM , (2.111)

where θ is a real parameter and Mˆ is an N × N .

a) Using the fact that the eigenvectors of an Hermitian matrix form a complete basis, find the spectrum (set of eigenvalues) and eigen- vectors of U in terms of the eigenvalues and eigenvectors in terms of Mˆ .

b) Using this result, prove that U is unitary. So far we have only considered rotations around the z axis of the Bloch sphere. By analogy with Eq. (2.96), we can rotate the state around the x axis through an angle χ via the unitary

−i χ σx Ux(χ) = e 2 , (2.112) around the y axis via

−i χ σy Uy(χ) = e 2 , (2.113) and around an arbitraryω ˆ axis via

−i χ ωˆ·⃗σ Uωˆ (χ) = e 2 . (2.114) Mathematically, the three components of the dimensionless spin angular mo- ⃗ 1 x y z mentum S = 2 (σ , σ , σ ) are the generators of (the of) rotations in spin space. As an example, let us consider a rotation by angle π/2 around the y axis to see how the cardinal points on the Bloch sphere transform into each other as expected: [ ] π π ˆ π y 1 Uy( )| + Z⟩ = cos I − i sin σ | ↑⟩ = √ [| ↑⟩ + | ↓⟩] 2 4 4 2 π U ( )| + Z⟩ = +| + X⟩, (2.115) y 2 π U ( )| − Z⟩ = −| − X⟩ (2.116) y 2 π U ( )| + X⟩ = +| − Z⟩ (2.117) y 2 π U ( )| − X⟩ = +| + Z⟩ (2.118) y 2 CHAPTER 2. INTRODUCTION TO HILBERT SPACE 38 where we have used the fact that ( )( ) ( ) 0 −i 1 0 −iσy| ↑⟩ = −i = = | ↓⟩ (2.119) +i 0 0 1 ( )( ) ( ) 0 −i 0 1 −iσy| ↓⟩ = −i = − = −| ↑⟩ (2.120) +i 0 1 0

Exercise 2.11. Prove that π U ( )| + Z⟩ = | − Y ⟩ (2.121) x 2 π U ( )| − Z⟩ = −i| + Y ⟩. (2.122) x 2

2.4.1 The Hadamard Gate We see from Eqs. (2.115-2.118) that a rotation by π/2 around the y axis interchanges the states ±Z and ±X, but not in a simple way. In order to swap the x and z components of the spin vector without the inconvenience of the various minus signs in Eqs. (2.116-2.118), quantum computer scientists like to invoke the Hadamard gate | ⟩⟨ | | − ⟩⟨− | H = + X +Z + X( Z ) (2.123) 1 1 1 1 = √ (σx + σz) = √ (2.124) 2 2 1 −1

Exercise 2.12. Using Eq. (2.123), prove that the Hadamard gate obeys

a) H = | + Z⟩⟨+X| + | − Z⟩⟨−X|,

b) H2 = I,

c) H is unitary.

2.5 Hilbert Space and Operators for Multiple Spins

So far we have been focused on the two-dimensional complex vector space describing the quantum states of a single spin. We turn now to the study CHAPTER 2. INTRODUCTION TO HILBERT SPACE 39 of how to extend this to two spins and ultimately to N spins. If we have a single spin, the standard basis has two states, | ↑⟩ and | ↓⟩ and the spin can be in any linear superposition of those two basis states. If we have two spins, we need four basis states: | ↓↓⟩, | ↓↑⟩, | ↑, ↓⟩, | ↑↑⟩. Replacing ↓ by 0 and ↑ by 1, we see that the states are labelled by the binary numbers corresponding to 0, 1, 2, 3 just as in a classical computer memory that contains only two bits. Our quantum bits can however be in a superposition of all four states of the form

|ψ⟩ = ψ↓↓| ↓↓⟩ + ψ↓↑| ↓↑⟩ + ψ↑↓| ↑↓⟩ + ψ↑↑| ↑↑⟩, (2.125) or equivalently a column vector of length four   ψ↑↑    ψ↑↓  |ψ⟩ =   . (2.126) ψ↓↑ ψ↓↓

Note that the choice of how to order the entries in the column is completely arbitrary, but once you make a choice you must stick to it for all your cal- culations. For the case of N spins, all of the above generalizes to a vector space of dimension 2N . We have so far seen two kinds of products for vectors, the inner product ⟨ψ|ϕ⟩ which is a scalar (complex number), and the outer product |ϕ⟩⟨ψ| which is an operator, that has a matrix representation. When it comes to thinking about the quantum states of a composite physical system consisting of multiple qubits, we have to yet another kind of product, the tensor product, sometimes called the direct product. The tensor product of the Hilbert space H1 of the first qubit with that of a second qubit H2 yields a new Hilbert space H1 ⊗ H2 whose dimension is the product of the dimensions of the two individual Hilbert spaces. The two-qubit basis states in Eq. (2.125) can be thought of as direct products of individual vectors,

| ↑↓⟩ = | ↑⟩ ⊗ | ↓⟩. (2.127)

The direct product of two column vectors of length n1 and n2 is a column vector of length n1 + n2. How do we represent operators in this multi-qubit space? First we have to put labels on the operators to denote which spin they are acting on. For CHAPTER 2. INTRODUCTION TO HILBERT SPACE 40 example, the zth component of the spin vector of the jth spin is denoted z σj , with j = 1, 2. Strictly speaking, we should write the tensor product z ⊗ 0 0 σ1 σ2, where σ2 is the identity operator acting on the second spin, but many authors take a shortcut in the notation by not explicitly writing out the identity operators. The tensor product of a matrix with dimension n1 × n1 with a matrix with dimension n2 × n2 is a larger matrix of dimension (n1 + n2) × (n1 + n2). This is of course consistent with the fact that the dimension of the direct product Hilbert space is n1 + n2. As an example consider the matrix representation of the operator ( ) ( ) +1 0 +1 0 σz ⊗ σ0 = ⊗ . (2.128) 1 2 0 −1 0 +1 The rule for how you write out the entries to this 4 × 4 matrix depend on exactly how you order the terms in the column vector in Eq. (2.126). It is clear however that we want the following to be true (assuming we number the spins in | ↑↓⟩ from left to right inside the Dirac ket) z ⊗ 0| ↑↑⟩ | ↑↑⟩ σ1 σ2 = (+1) (2.129) z ⊗ 0| ↑↓⟩ | ↑↓⟩ σ1 σ2 = (+1) (2.130) z ⊗ 0| ↓↑⟩ − | ↓↑⟩ σ1 σ2 = ( 1) (2.131) z ⊗ 0| ↓↓⟩ − | ↓↓⟩ σ1 σ2 = ( 1) , (2.132) and 0 ⊗ z| ↑↑⟩ | ↑↑⟩ σ1 σ2 = (+1) (2.133) 0 ⊗ z| ↑↓⟩ − | ↑↓⟩ σ1 σ2 = ( 1) (2.134) 0 ⊗ z| ↓↑⟩ | ↓↑⟩ σ1 σ2 = (+1) (2.135) 0 ⊗ z| ↓↓⟩ − | ↓↓⟩ σ1 σ2 = ( 1) . (2.136) Using the basis implicit in Eq. (2.126), the outer product in Eq. (2.128) is represented by the matrix   +1 0 0 0  0 +1 0 0  σz ⊗ σ0 =   . (2.137) 1 2  0 0 −1 0  0 0 0 −1 We can think of the direct product as producing a 2×2 array of 2×2 identity z matrices, each multiplied by one of the four entries in σ1. Reversing the order CHAPTER 2. INTRODUCTION TO HILBERT SPACE 41 of the operators gives   +1 0 0 0  0 −1 0 0  σ0 ⊗ σz =   . (2.138) 1 2  0 0 +1 0  0 0 0 −1 We can think of this as four copies of the σz matrix each multiplied by the appropriate entry in the identity matrix. We can also consider the direct sum of two operators. For example, the z component of the total spin angular momentum is given by z ≡ z ⊕ z S σ1 σ2, (2.139) is represented in matrix form by   +2 0 0 0  0 0 0 0  Sz = σz ⊗ σ0 + σ0 ⊗ σz =   . (2.140) 1 2 1 2  0 0 0 0  0 0 0 −2

In Wolfram Mathematica⃝R , the following commands will produce these direct products (called outer products in Mathematica⃝R ) II = IdentityMatrix[2]; MatrixForm[II] Z = {{1, 0}, {0, -1}}; MatrixForm[Z] ZI = ArrayFlatten[Outer[Times, Z, II]]; MatrixForm[ZI] IZ = ArrayFlatten[Outer[Times, II, Z]]; MatrixForm[IZ] MatrixForm[IZ + ZI]

Exercise 2.13. Write out the matrix representation of the following two-qubit Pauli matrices x ⊗ x a) σ1 σ2 . x ⊕ x b) σ1 σ2 . ( ) ( ) 0 z ⊕ 1 0 z c) σ1 + σ1 2 σ2 + σ2 . CHAPTER 2. INTRODUCTION TO HILBERT SPACE 42

Box 2.3. The No-cloning Theorem The no-cloning theorem states that it is impossible to make a copy of an unknown quantum state. The essential idea of the no-cloning theorem is that in order to make a copy of an unknown quantum state, you would have to measure it to see what the state is and then use that knowledge to make the copy. However measurement of the state produces random back action (state collapse) and it is not possible to fully determine the state. This is a reflection of the fact that measurement of a qubit yields one classical bit of information which is not enough in general to fully specify the state via its latitude and longitude on the Bloch sphere. Of course if you have prior knowledge, such as the fact that the state is an eigenstate of σx, then a measurement of σx tells you the eigenvalue and hence the state. The measurement gives you one additional classical bit of information which is all you need to have complete knowledge of the state. A more formal statement of the no-cloning theorem is the following. Give an unknown state |ψ⟩ = α| ↑⟩ + β| ↓⟩ and an ancilla qubit initially prepared in a definite state (e.g. | ↓⟩), there does not exist a unitary operation U that will take the initial state

|Φ⟩ = [α| ↑⟩ + β| ↓⟩] ⊗ | ↓⟩ (2.141)

to the final state

U|Φ⟩ = [α| ↑⟩ + β| ↓⟩] ⊗ [α| ↑⟩ + β| ↓⟩] , = α2| ↑↑⟩ + αβ[| ↑↓⟩ + | ↓↑⟩] + β2| ↓↓⟩. (2.142)

unless U depends on α and β. The proof is straightforward. The RHS of Eq. (2.141) is linear in α and β, whereas the RHS of Eq. (2.142) is quadratic. This is impossible unless U depends on α and β. For an unknown state, we do not know α and β and therefore cannot construct U.

Exercise 2.14. Assuming you have knowledge of α and β, construct an explicit unitary U(α, β) that carries out the cloning operation in Eq. (2.142). Chapter 3

Two-Qubit Gates and Entanglement

3.1 Introduction

We learned in Chap. 2 that the physical operations we are allowed to carry out (besides measurement) are represented by unitary matrices acting on state vectors. These operations are rotations in Hilbert space and transform one state into another. Indeed, this is all a quantum computer can do. You load a quantum state into the N qubits of the input register, carry out a unitary rotation in the 2N -dimensional Hilbert space, and place the resulting state into the N qubits of the output register. Measure the state of the output register and you acquire N classical bits of information.1 Unitary operations can be combined with measurements in interesting ways. After carrying out a unitary U0, one could measure a subset of say m qubits. The measurement will yield m classical bits of information corresponding to one instance out of 2m possible results. Conditioned on this, one can choose which m of 2 subsequenty unitaries Uj from the set U1,U2,...,U2m to apply. The net effect of these three steps is the most general possible quantum operation

1As noted earlier, N classical bits is far less information than it takes to specify a generic N-qubit state. Thus generically the measurements give random results as the measured state collapses. However certain algorithms, e.g. Shor’s factoring algorithm use quantum interference to ‘focus’ the amplitudes in the output state that it consists of a superposition of only a small number of states in the standard basis and thus the result is not very random. Repeating the algorithm a small number of times yields the desired prime factors with high probability.

43 CHAPTER 3. TWO-QUBIT GATES AND ENTANGLEMENT 44 and is not equivalent to a unitary transformation. Such quantum operations can be more powerful for certain purposes (e.g. quantum error correction). For now however, let us concentrate on two-qubit gates (unitary opera- tions). We saw in Chap. 2 that Hilbert space consisting of the direct product of the Hilbert space for two single qubits is four-dimensional and that oper- ators on that space are represented by 4 × 4 matrices. It does not seem that stepping up from one qubit to two qubits is going to lead to anything dra- matically new. However, it does–we will explore the fascinating concept of entanglement. Entanglement was deeply disturbing to the founders of quan- tum mechanics, especially to Einstein who understood and pointed out its spooky implications. The power of entanglement as a quantum resource has come to be appreciated in recent decades and underlies certain key protocols for quantum communication and for quantum computation.

3.2 The CNOT Gate

The NOT gate in a classical computer simply flips the bit taking 0 to 1 and 1 to 0. The so-called ‘controlled-NOT’ or CNOT gate allows one to do ‘if- then-else’ logic with a computer. It is a two-bit gate that flips the state of the target bit conditioned on the state of the control qubit. It has the ‘truth table’ shown in Table 3.1 from which we see that the target bit is flipped, if and only if, the control bit is in the 1 state.

CNOT Truth Table Control In Target In Control Out Target Out 0 0 0 0 0 1 0 1 1 0 1 1 1 1 1 0

Table 3.1: Action of the two-bit CNOT gate in the standard basis.

In the classical context one can imagine measuring the state of the control bit and then using that information to control the flipping of the target bit. However in the quantum context, it is crucially important to emphasize that measuring the control qubit would collapse its state. We must therefore avoid any measurements and seek a unitary gate which works correctly when CHAPTER 3. TWO-QUBIT GATES AND ENTANGLEMENT 45 the control qubit is in |0⟩ and |1⟩ and even when it is in a superposition of both possibilities α|0⟩ + β|1⟩. As we will soon see, it is this latter situation which will allow us to generate entanglement. When the control qubit is in a superposition state, the CNOT gate causes the target qubit to be both flipped and not flipped in a manner that is correlated with the state of the control qubit. As we will see, these are not ordinary classical statistical correlations (e.g. clouds are correlated with rain), but rather special (and powerful) quantum correlations resulting from entanglement.

Box 3.1. CNOT without Measurement How can we possibly flip the state of the target qubit conditioned on the state of the control qubit without measuring the control and hence collapsing its state? We know that in many systems we can cause a transition between two quantum levels separated in energy by an amount ~ω by applying an electromagnetic wave of frequency ω for a certain precise period of time. The way to make this bit flip of the target be conditioned on the state of the control is to have an interaction between the two qubits that causes the transition energy of the target depend on the state of the control. For example, consider the Hamiltonian (the energy operator)

~ω ~ω H = 1 Z + 2 Z + ~gZ Z . (3.1) 2 1 2 2 1 2

The energy change to flip qubit 2 from | ↓⟩ to | ↑⟩ is ∆E = ~(ω2 + gZ1). Thus if we shine use light of frequency ~(ω2 + g), qubit 2 will flip when qubit 1 is in | ↑⟩ because that matches the transition frequency. However if qubit 1 is in the state | ↓⟩, the transition frequency for qubit 2 is shifted to ~(ω2 − g) and the light has the wrong frequency to cause the transition. See Fig. 3.1 for an illustration of the level scheme.

From the classical truth table we can attempt to construct the appropriate quantum operator by putting the dual of initial state ket in the bra and the desired final state in the ket

CNOT = |00⟩⟨00| + |01⟩⟨01| + |11⟩⟨10| + |10⟩⟨11|. (3.2)

We see that this produces all the desired transformations, for example

CNOT|11⟩ = |10⟩. (3.3) CHAPTER 3. TWO-QUBIT GATES AND ENTANGLEMENT 46

|nn²

Z2  2g

Z1  2g

|np²

|pn²

Z1  2g

Z2  2g |pp²

Figure 3.1: Energy levels for two interacting qubits whose Hamiltonian is given by Eq. (3.1). Solid lines correspond to transitions in which qubit 1 is flipped. Dashed lines correspond to transitions in which qubit 2 is flipped. By choosing ω1, ω2 and g appropriately, all four single-spin transition frequencies can be made unique, thereby allowing flip of one qubit conditioned on the state of the other. If qubit 1 is the control and qubit 2 is the target, then illumination at frequency ω2 + 2g executes the CNOT. Illumination at frequency ω1 + 2g executes a CNOT with qubit 2 being the control and qubit 1 being the target. If g = 0, then the transition energy for one qubit is not dependent on the state of the other and conditional operations are not possible.

This however is not enough. We have to prove that the desired transforma- tions are legal. That is, we must show that CNOT is unitary. It is clear by inspection that CNOT is Hermitian. A straightforward calculation shows that (CNOT)2 = I. Hence by the lemma in Ex. 2.9b, CNOT is unitary. It is also instructive to write the gate in the following manner (dispensing with the ⊗ symbol and number the qubits 1 and 2 from left to right) ( ) ( ) I + Z I − Z CNOT = 1 1 X + 1 1 I . (3.4) 2 2 2 2 The first parentheses enclose the projector onto the up state for the control qubit (qubit 1). The second parentheses enclose the projector onto the down state for the control qubit. Thus if qubit 1 is in | ↑⟩ = |1⟩, then X2 flips the second qubit while the remaining term vanishes. Conversely when qubit 1 is in | ↓⟩ = |0⟩, the coefficient of X2 vanishes and only the identity in the second term acts. In the standard two-qubit basis defined in Eq. (2.126), the operator in CHAPTER 3. TWO-QUBIT GATES AND ENTANGLEMENT 47

Eq. (3.2) and Eq. (3.4) has the matrix representation

Box 3.2. Some desired operations are not unitary. For example, one important task in a quantum computer is to reset all the bits to some standard state before starting a new computation. Let us take the standard state to be |0⟩. We want to map every initial state to |0⟩ which can be done with the following operator ( ) 0 0 R = |0⟩⟨0| + |0⟩⟨1| = . (3.5) 1 1

We see immediately that R cannot be unitary because a unitary operation preserves inner products between states. The two initial states |0⟩ and |1⟩ are orthogonal and yet they both map to the state |0⟩ which means that the states under the mapping are no longer orthogonal. Equivalently, we see that R is not invertible because we don’t know if R−1 is supposed to take |0⟩ to |0⟩ or to |1⟩. One can also mechanically check that R is not unitary by writing

R† = |0⟩⟨0| + |1⟩⟨0|, (3.6) † | ⟩⟨ | | ⟩⟨ | | ⟩⟨ | | ⟩⟨ | R R = ( 0 0 +)1 0 )( 0 0 + 0 1 ) (3.7) 1 1 = = I + σx ≠ I. (3.8) 1 1 Thus if we only have access to unitary rotations, we cannot reset an unknown state. (Of course if we knew the state, we could find a rotation that would take it back to 0⟩.) There is however a simple fix for this problem. If we measure σz (say), the unknown state will randomly collapse to either |0⟩ or |1⟩. If it collapses to |0⟩ apply the identity operation. If it collapses to |1⟩, then apply the unitary operation σx to flip the state from |1⟩ to |0⟩. We see that applying one of two different unitaries (I or σx) conditioned on the outcome of a measurement gives us a new capability beyond that of unitary operations. CHAPTER 3. TWO-QUBIT GATES AND ENTANGLEMENT 48

( ) ( ) ( ) ( ) 1 0 0 1 0 0 1 0 CNOT = ⊗ + ⊗ (3.9) 0 0 1 0 0 1 0 1   0 1 0 0  1 0 0 0  =   . (3.10)  0 0 1 0  0 0 0 1

The CNOT unitary is represented in ‘’ notation by the construction illustrated in Fig. 3.2

| q1²

| q2 ²

Figure 3.2: Quantum circuit representation of the CNOT operation. The filled cir- cle denotes the control qubit (in this case, q1) and the symbol ⊕ denotes the target qubit (in this case, q2) for the gate. In quantum circuit notation the order in which gates are applied (‘time’) runs from left to right. If the circuit has GATE1 followed by GATE2, reading from left to right, this corresponds to the (right-to-left) sequence GATE2 GATE1|INPUT STATE⟩.

Exercise 3.1. Consider the reset operator R defined in Box 3.2. Find a state whose norm is not preserved under R. This is further proof that R is not a legal unitary operator and requires the assistance of a measurement operation. Now that we have established the CNOT unitary and how it acts on the standard basis, let us consider what happens when the control qubit is in a superposition state. We begin with a so-called product state or separable state which can be written as the product of a state for the first qubit and a state for the second qubit

|Ψ⟩ = [α1|0⟩ + β1|1⟩] ⊗ [α2|0⟩ + β2|1⟩]

= α1α2|00⟩ + α1β2|01⟩ + β1α2|10⟩ + β1β2|11⟩. (3.11) CHAPTER 3. TWO-QUBIT GATES AND ENTANGLEMENT 49

For example consider the following separable initial state 1 |ψ⟩ = | − X⟩| + Z⟩ = | ←⟩| ↑⟩ = √ (| ↑⟩ − | ↓⟩) | ↓⟩ (3.12) 2 1 = √ (| ↑↑⟩ − | ↓↑⟩) . (3.13) 2 When we apply the CNOT gate to this state, the target qubit both flips and doesn’t flip! The two qubits become ‘entangled’ 1 |B0⟩ = CNOT|ψ⟩ = √ (| ↑↓⟩ − | ↓↑⟩) . (3.14) 2 An entangled state of two qubits2 is any state which cannot be written in separable form. A convenient basis in which to represent entangled states is the so-called Bell basis3 1 |B0⟩ = √ [| ↑↓⟩ − | ↓↑⟩] (3.15) 2 1 |B1⟩ = √ [| ↑↓⟩ + | ↓↑⟩] (3.16) 2 1 |B2⟩ = √ [| ↑↑⟩ − | ↓↓⟩] (3.17) 2 1 |B3⟩ = √ [| ↑↑⟩ + | ↓↓⟩] . (3.18) 2 Each of these four states is a ‘maximally entangled’ state, but they are mu- tually orthogonal and therefore must span the full four-dimensional Hilbert space. Thus linear superpositions of them can represent any state, including even product states. For example, 1 | ↑⟩| ↑⟩ = | ↑↑⟩ = √ [|B2⟩ + |B3⟩] . (3.19) 2 2The concept of entanglement is more difficult to uniquely define and quantify for N > 2 qubits. 3Named after John Bell, the physicist who in the 1960’s developed deep insights into the issues surrounding the concept of entanglement that so bothered Einstein. Bell proposed a rigorous experimental test of the idea that the randomness in quantum experiments is due to our inability to access hidden classical variables. At the time this was a theorist’s ‘gedanken’ experiment, but today the ‘’ has rigorously ruled out the possibility of hidden variables. Indeed, the Bell test is now a routine engineering test to make sure that your quantum computer really is a quantum computer, not a classical computer. CHAPTER 3. TWO-QUBIT GATES AND ENTANGLEMENT 50

Entanglement is very mysterious and entangled states have many pecu- liar and counter-intuitive properties. In an entangled state the individual spin components have zero expectation value and yet the spins are strongly correlated. For example,

⟨B0|⃗σ1|B0⟩ = ⃗0 (3.20)

⟨B0|⃗σ2|B0⟩ = ⃗0 (3.21) ⟨ | x x| ⟩ − B0 σ1 σ2 B0 = 1 (3.22) ⟨ | y y| ⟩ − B0 σ1 σ2 B0 = 1 (3.23) ⟨ | z z| ⟩ − B0 σ1σ2 B0 = 1. (3.24) This means that the spins have quantum correlations which are stronger than is possible classically. In particular,

⟨B0|⃗σ1 · ⃗σ2|B0⟩ = −3 (3.25) despite the fact that in any single-spin (or product state) |⟨⃗σ⟩| = 1. We can get obtain a useful picture of the Bell states by examining all of the possible two-spin correlators in the so-called ‘Pauli bar plot’ for the state |B0⟩ shown in Fig. (3.3). We see that all the single-spin operator (e.g. IX and ZI) expectation values vanish. Because of the entanglement, each spin is, on average, totally unpolarized. Yet three of the two-spin correlators, XX,YY,ZZ, are all -1 indicating that the two spins are pointing in exactly opposite directions. This is becasue |B0⟩ is the rotationally invariant ‘spin- singlet’ state.

1.0

0.5

II IX IY IZ XI YI ZI XX XY XZ YX YY YZ ZX ZY ZZ

-0.5

-1.0

Figure 3.3: ‘Pauli bar plot’ of one and two spin correlators in the Bell state |B0⟩. CHAPTER 3. TWO-QUBIT GATES AND ENTANGLEMENT 51

In maximally entangled states Bell states, the individual spins do not have their own ‘state’ in the sense that ⟨B|⃗σ1|B⟩ = ⃗0, rather thann ˆ, where nˆ is some unit vector. We can only say for |B0⟩ for example that the z components of the two spins will always be measured to be exactly opposite each other and have product −1. Remarkably, the same is true for the x and y components as well. Classical vectors of length 1 drawn from a random simply cannot have this property. Thus the correlations of the spins in entangled states are intrinsically quantum in nature and stronger than is possible in any classical (or ‘hidden variable’ quantum) model. Exercise 3.2. Derive Eqs. (3.20-3.25)

3.3 Bell Inequalities

We turn now to further consideration of the ‘spooky’ correlations in entan- gled states. We have already seen for the Bell state |B0⟩ that the components of the two spins are perfectly anti-correlated. Suppose now that Alice pre- pares two qubits in this Bell state and then sends one of the two qubits to Bob who is far away (say one light-year). Alice now chooses to make make a measurement of her qubit projection along some arbitrary axisn ˆ. For sim- plicity let us say that she chooses thez ˆ axis. Let us further say that her measurement result is −1. Then she immediately knows that if Bob chooses to measure his qubit along the same axis, his measurement result will be the opposite, +1. It seems that Alice’s measurement has collapsed the state of the spins from |B0⟩ to | ↓↑⟩. This ‘spooky action at a distance’ in which Al- ice’s measurement seems to instantaneously change Bob’s distant qubit was deeply troubling to Einstein [2]. Upon reflection one can see that this effect cannot be used for superlu- minal communication (in violation of special relativity). Even if Alice and Bob had agreed in advance on what axis to use for measurements, Alice has no control over her measurement result and so cannot use it to signal Bob. It is true that Bob can immediately find out what Alice’s measurement result was, but this does not give Alice the ability to send a message. In fact, suppose that Bob’s clock were off and he accidentally made his mea- surement slightly before Alice. Would either he or Alice be able to tell? The answer is no, because each would see a random result just as expected. This CHAPTER 3. TWO-QUBIT GATES AND ENTANGLEMENT 52 must be so because in special relativity, the very concept of simultaneity is frame-dependent and not universal. Things get more interesting when Alice and Bob choose different mea- surement axes. Einstein felt that quantum mechanics must somehow not be a complete description of reality and that there might be ‘hidden variables’ which if they could be measured would remove the probabilistic nature of the quantum description. However in 1964 John S. Bell proved a remarkable inequality [3] showing that when Alice and Bob use certain different measure- ment axes, the correlations between their measurement results are stronger than any possible classical correlation that could be induced by (local) hidden variables. Experimental violation of the Bell inequality proves that it is not true that quantum observables have values (determined by hidden classical variables) before we measure them. Precision experimental measurements which violate the Bell inequalities are now the strongest proof that quantum mechanics is correct and that local hidden variable theories are excluded. Perhaps the simplest way to understand this result is to consider the CHSH inequality developed by Clauser, Horn, Shimoni and Holt [4] follow- ing Bell’s ideas. Consider the measurement axes shown in Fig. (3.4). The experiment consists of many trials of the following protocol. Alice and Bob share a pair of qubits in an entangled state. Alice randomly chooses to mea- sure the first qubit using X or Z while Bob randomly chooses to measure the second qubit using X′ or Z′ which are rotated 45 degrees relative to Alice’s axes. After many trials (each starting with a fresh copy of the entangled state), Alice and Bob compare notes (via classical subluminal communica- tion) on their measurement results and compute the following correlation function

S = ⟨XX′⟩ + ⟨ZZ′⟩ − ⟨XZ′⟩ + ⟨ZX′⟩. (3.26)

By correlation function of two measurement results we mean (for example)

′ ′ ⟨XZ ⟩ = ⟨B0|XZ |B0⟩. (3.27)

Experimentally this is measured by the following. For the XZ′ correlator, Alice and Bob select those instances in which Alice happens to have chosen to measure X and Bob happens to have chosen to measure Z′. (This occurs say N times, corresponding to 25% of the total number of trials on average.) ± ′ ± Let xj = 1 and zj = 1 be respectively Alice’s random measurement result for X in the jth trial and and Bob’s random measurement result for Z′ in the CHAPTER 3. TWO-QUBIT GATES AND ENTANGLEMENT 53 jth trial. Then Alice and Bob’s comparision of measurement results can be used to create the following unbiased (but noisy) estimator of the correlation

1 ∑N ⟨B |XZ′|B ⟩ ≈ x z . (3.28) 0 0 N j j j=1

In the limit of large N the statistical uncertainty in the estimator goes to zero and we obtain an arbitrarily accurate estimate of the correlation. The four possible combinations of the two measurement results are illustrated in Fig. 3.5. The correlator is given by the probabilities of the four outcomes

′ ⟨B0|XZ |B0⟩ = P++ − P−+ + P−− − P+−. (3.29)

′ If the measurements are perfectly correlated (xj = zj every time) then the − ′ correlator will be +1. If perfectly anticorrelated (xj = zj every time), then the correlator will be −1. If the measurements are uncorrelated, then all four ′ measurement outcomes will be equally likely and xjzj will be fully random (±1 with equal probability) and the correlator will vanish (on average).

Z

Z¢ X ¢

o 45 X

Figure 3.4: Measurement axes used by Alice (solid lines) and Bob (dashed lines) in estab- lishing the Clauser-Horn-Shimoni-Holt (CHSH) inequality.

Eq. (3.26) can be rewritten

S = ⟨(X + Z)X′⟩ − ⟨(X − Z)Z′⟩. (3.30)

Alice and Bob note that their measurement results are random variables which are always equal to either +1 or −1. In a particular trial Alice chooses randomly to measure either X or Z. If you believe in the hidden variable theory, then surely, the quantities not measured still have a value of either CHAPTER 3. TWO-QUBIT GATES AND ENTANGLEMENT 54

zcj ( 1,  1) ( 1,  1)

x j

( 1,  1) ( 1,  1)

Figure 3.5: Illustration of the four possible measurement outcomes in the jth run of an experiment in which Alice measures X and Bob measures Z′. The net correlator of the two measurements is given by Eq. (3.29).

+1 or −1 (because when we do measure them, they always are either +1 or −1). If this is true, then either X = Z or X = −Z. Thus either X + Z vanishes or X − Z vanishes in any given realization of the random variables. The combination that does not vanish is either +2 or −2. Hence it follows immediately that S is bounded by the CHSH inequality

−2 ≤ S ≤ +2. (3.31)

It turns out however that the quantum correlations between the two spins in the entangled pair violate this classical bound. They are stronger than can ever be possible in any classical local hidden variable theory. To see this, note that because ⃗σ is a vector, we can resolve its form in one basis in terms of the other via

′ 1 σx = √ [σz + σx] (3.32) 2 ′ 1 σz = √ [σz − σx]. (3.33) 2 Thus we can express S in terms of the ‘Pauli bar’ correlations 1 S = √ [⟨XX + XZ + ZX + ZZ⟩ − ⟨XZ − XX − ZZ + ZX⟩] . (3.34) 2

For Bell state |B0⟩, these correlations are shown in Fig. (3.3) and yield √ S = −2 2, (3.35) CHAPTER 3. TWO-QUBIT GATES AND ENTANGLEMENT 55 in clear violation of the CHSH inequality. Strong violations of the CHSH inequality are routine in modern experiments.4 This teaches us that in our quantum world, observables do not have values if you do not measure them. There are no hidden variables which determine the random values. The unobserved spin components simply do not have values. Recall that σx and σz are incompatible observables and when we choose to measure one, the other remains not merely unknown, but unknowable. It is ironic that Einstein’s arguments that quantum mechanics must be incomplete because of the spooky properties of entanglement have led to the strongest experimental tests verifying the quantum theory and falsifying all local hidden variable theories. We are forced to give up the idea that physical observables have values before they are observed. Exercise 3.3. Other Bell inequalities.

1. Work out the ‘Pauli bar plots’ (analogous to Fig. 3.3) for each of the Bell states B1,B2,B3. 2. Using the same quantization axes as in Fig. 3.4, find the analog of Eq. (3.26) for the correlators that should be measured to achieve violation of the Bell inequality for these other Bell states.

3.4 Quantum Dense Coding

Now that we have learned about Bell states describing the entanglement of a pair of qubits, we will show that entanglement is a powerful resource for quantum information tasks. In this section we will see how can be used as a resource to help in the task of quantum communication. We saw above that quantum correlations are strong enough to violate certain classical bounds, but the spooky action at a distance seemed unable to help us send signals. Actually it turns out that by using a special ‘quan- tum dense coding’ protocol, we can use entanglement to help us transmit

4Although strictly speaking, in many experiments there are loopholes associated with imperfections in the detectors and the fact that Alice and Bob typically do not have a space-like separation. However recent experiments have closed all these loopholes. Note that all measured correlators have statistical uncertainty in them when the number of measurements N is finite. However modern experiments can achieve results that exceed the Bell bound by many standard deviations. CHAPTER 3. TWO-QUBIT GATES AND ENTANGLEMENT 56 information in a way that is impossible classically. [1] As background let us recall that for a single qubit there are only two orthogonal states which are connected by a rotation of the spin through an angle of π. For rotation by π around the y axis the unitary rotation operator is

y −i π σy y 2 − Rπ = e = iσ , (3.36) and we map between the two orthogonal states y | ↑⟩ | ↓⟩ Rπ = + (3.37) y | ↓⟩ −| ↑⟩ Rπ = (3.38) while any other rotation angle simply produces general linear combinations of the two basis states. For example, for rotation by π/2 we have

y −i π σy 1 y R π = e 4 = √ [1 − iσ ], (3.39) 2 2 and we have

y 1 R π | ↑⟩ = √ [| ↑⟩ + | ↓⟩] = | →⟩ (3.40) 2 2 y 1 R π | ↓⟩ = −√ [| ↑⟩ − | ↓⟩] = −| ←⟩. (3.41) 2 2 The rotated states are linearly independent of the starting state but never orthogonal to it except for the special case of rotation by π. The situation is very different for two-qubit entangled states. We take as our basis the four orthogonal Bell states. Suppose that Alice prepares the Bell state |B0⟩ and sends one of the qubits to Bob who is in a distant location. Using a remarkable protocol called quantum dense coding [1], Alice can now send Bob two classical bits of information by sending him the remaining qubit. The protocol relies on the amazing fact that Alice can transform the initial Bell state into any of the others by purely local operations on her remaining qubit without communicating with Bob. The four possible unitary CHAPTER 3. TWO-QUBIT GATES AND ENTANGLEMENT 57

5 operations Alice should perform are I1, −X1, −iY1,Z1 which yield

I1|B0⟩ = |B0⟩ (3.42)

Z1|B0⟩ = |B1⟩ (3.43)

−X1|B0⟩ = |B2⟩ (3.44)

−iY1|B0⟩ = |B3⟩. (3.45) It seems somehow miraculous that without touching Bob’s qubit, Alice can reach all four orthogonal states by merely rotating her own qubit. How- ever the fact that this is possible follows immediately from the Pauli bar plot in Fig. 3.3 and the corresponding plots for the other Bell states. In every Bell state, the expectation value of every component of ⃗σ1 (and ⃗σ2) vanishes. Thus for example

⟨ | x| ⟩ ⟨ | | ⟩ B0 σ1 B0 = B0 X1 B0 = 0. (3.46) x| ⟩ | ⟩ But this can only occur if the state σ1 B0 is orthogonal to B0 ! This in turn means that there are four possible two-bit messages Alice can send by associating each with one of the four operations I1,Z1, −X1, −iY1 according to the following6 message operation 00 −X1 01 I1 10 −iY1 11 Z1 After encoding her message by carrying out the appropriate operation on her qubit, Alice physically transmits her qubit to Bob. Bob then makes a joint measurement (details provided further below) on the two qubits which tells him which of the four Bell states he has and thus recovers two classical bits of information even though Alice sent him only one quantum bit after deciding what the message was. The pre-positioning of the entangled pair

5 As usual we are simplifying the notation. For example Z1 stands for the more math- ematically formal Z ⊗ I since it applies Z to the first (Alice’s) qubit and the identity to the second (Bob’s) qubit. Note that the phase factor -1 in front of X1 and −i in front of Y1 are irrelevant global phases. 6The particular operators associated with each of the four binary numbers is somewhat arbitrary and was chosen in this case to correspond to a particular choice of Bob’s decoding circuit which will be described later. CHAPTER 3. TWO-QUBIT GATES AND ENTANGLEMENT 58 has given them a resource which doubles the number of classical bits that can be transmitted with one (subsequent) quantum bit. Of course in total Alice transmitted two qubits to Bob. The key point is that the first one was sent in advance of deciding what the message was. How weird is that!? This remarkable protocol sheds considerable light on the concerns that Einstein raised in the EPR paradox [2]. It shows that the special correlations in Bell states can be used to communicate information in a novel and efficient way by ‘prepositioning’ entangled pairs shared by Alice and Bob. However causality and the laws of special relativity are not violated because Alice still has to physically transmit her qubit(s) to Bob in order to send the information. The above protocol requires Bob to determine which of the four Bell states he has. The quantum circuit shown in Fig. (3.6) uniquely maps each of the Bell states onto one of the four computational basis states (eigenstates of Z1 and Z2). The first symbol indicates the CNOT gate which flips the target qubit (in this case qubit 2) if and only if the control qubit (qubit 1) is in the excited state. The second gate in the circuit (denoted H) is the Hadamard gate that was introduced in Sec. 2.4.1 acting on the first qubit. H

Figure 3.6: Bell measurement circuit with a CNOT and Hadamard gate. This circuit permits measurement of which Bell state a pair of qubits is in by mapping the states to z z the basis of eigenstates of σ1 and σ2 .

Exercise 3.4. Prove the following identities for the circuit shown in Fig. (3.6) where qubit 1 is the control and qubit 2 is the target

H1 CNOT12|B0⟩ = +|01⟩ (3.47)

H1 CNOT12|B1⟩ = +|11⟩ (3.48)

H1 CNOT12|B2⟩ = +|00⟩ (3.49)

H1 CNOT12|B3⟩ = +|10⟩ (3.50) Once Bob has mapped the Bell states onto unique computational basis CHAPTER 3. TWO-QUBIT GATES AND ENTANGLEMENT 59

states, he measures Z1 and Z2 separately, thereby gaining two bits of classical information and effectively reading the message Alice has sent. Note that the overall sign in front of the basis states produced by the circuit is irrelevant and not observable in the measurement process. Also note that to create Bell states in the first place, Alice can simply run the circuit in Fig. (3.6) backwards. Exercise 3.5. Construct explicit quantum circuits that take the start- ing state |00⟩ to each of the four Bell states.)

3.5 No-Cloning Theorem Revisited

Now that we have understood the EPR paradox and communication via quantum dense coding, we can gain some new insight into the no-cloning theorem. It turns out that if cloning of an unknown quantum state were possible, then we could use an entangled pair to communicate information at superluminal speeds in violation of special relativity. Consider the following protocol. Alice and Bob share an entangled Bell pair in state |B0⟩. Alice chooses to measure her qubit in either the Z basis or the X basis. The choice she makes defines one classical bit of information. The result of the measurement collapses the entangled pair into a simple product state. If Alice chooses the Z basis for her measurement, then Bob’s qubit will be either | ↑⟩ or | ↓⟩. If Alice chooses to measure in the X basis, then Bob’s qubit will be either | →⟩ or | ←⟩. Bob can distinguish these cases by cloning his qubit to make many copies. If he measures a large number of copies in the Z basis and always gets the same answer, he knows that his qubit is almost certainly in a Z eigenstate. If even a single measurement result is different from the first, he knows his qubit cannot be in a Z eigenstate and so must be in an X eigenstate. (Of course he could also measure a bunch of copies in the X basis and gain the same information.) This superluminal communication would violate special relativity and hence cloning must be impossible. In fact, cloning would make it possible for Alice to transmit an unlimited number of classical bits using only a single Bell pair. Alice could choose an arbitrary measurement axisn ˆ. The specification ofn ˆ requires two real num- bers (the polar and azimuthal angles). It would take a very large number of bits to represent these real numbers to some high accuracy. Now if Bob can make an enormous number of copies of his qubit, he can divide the copies in three groups and measure the vector spin ⟨⃗σ⟩ to arbitrary CHAPTER 3. TWO-QUBIT GATES AND ENTANGLEMENT 60 accuracy. From this he knows the polarizaton axisn ˆ = ±⟨⃗σ⟩ Alice chose (up to an unknown sign since he does not know the sign of Alice’s measurement result forn ˆ · σ). Hence Bob has learned a large number of classical bits of information. The accuracy (and hence the number of bits) is limited only by the statistical uncertainties resulting from the fact that his individual mea- surement results can be random, but these can be reduced to an arbitrarily low level with a sufficiently large number of copies of the state.

3.6 Quantum Teleportation

[Not covered in class in 2019] Even though it is impossible to clone an unknown quantum state, it is possible for Alice to ‘teleport’ an unknown state to Bob as long as her copy of the original is destroyed in the process [5]. Just as for quantum dense coding, teleportation protocols take advantage of the power of ‘pre- positioned’ entangled pairs. However unlike quantum dense coding where Alice ultimately sends her qubit to Bob, teleportation only requires Alice to send two classical bits to Bob. She does not have to send any additional quantum bits to Bob after pre-positioning the initial Bell pair. A simple protocol is as follows: Alice creates a |B0⟩ Bell state and sends one of the qubits to Bob. Alice has in her possession an additional qubit in an unknown state

|ψ⟩ = α| ↑⟩ + β| ↓⟩ (3.51) which she wishes Bob to be able to obtain without her sending it to him. Again, this must done at the expense of destroying the state of her copy of the qubit because of the no-cloning theorem. Alice applies the Bell state determination protocol illustrated in Fig. (3.6) to determine the joint state of the unknown qubit and her half of the Bell pair she shares with Bob. She then transmits two classical bits giving her measurement results to Bob. To see how Bob is able to reconstruct the initial state, note that we can rewrite the initial state of the three qubits in the basis CHAPTER 3. TWO-QUBIT GATES AND ENTANGLEMENT 61 of Bell states for the two qubits that Alice will be measuring as follows

|Ψ⟩ = [α| ↑⟩ + β| ↓⟩]|B0⟩ (3.52) α β = √ [| ↑↑↓⟩ − | ↑↓↑⟩] + √ [| ↓↑↓⟩ − | ↓↓↑⟩] (3.53) 2 2 1 = |B ⟩[−α| ↑⟩ − β| ↓⟩] 2 0 1 + |B ⟩[−α| ↑⟩ + β| ↓⟩] 2 1 1 + |B ⟩[−β| ↑⟩ − α| ↓⟩] 2 2 1 + |B ⟩[+β| ↑⟩ − α| ↓⟩] (3.54) 2 3 From this representation we see that when Alice tells Bob which Bell state she found, Bob can find a local unitary operation to perform on his qubit to recover the original unknown state (up to an irrelevant overall sign). The appropriate operations are Alice’s Bell state Bob’s operation |B0⟩ I |B1⟩ Z |B2⟩ X |B3⟩ Y Algebra leading to this table needs to be checked.

3.7 YET TO DO:

1. Explain how Hadamard gates interchange control and target in the CNOT.

2. Show that there is NO state for which Alice can change the probability of the outcomes for Bob’s Z measurement by applying any unitary to her qubit. (Because operators on different qubits commute.) However Alice can change Bob’s probability by making a measurement of her qubit. But, she can’t control the outcome of the measurement, so there is no superluminal communication. CHAPTER 3. TWO-QUBIT GATES AND ENTANGLEMENT 62

3. The value measured for the observable is always one of the eigenvalues of that observable and the state always collapses to the corresponding eigenvector. (If two or more of the eigenvalues are degenerate, then the situation is slightly more subtle. The state is projected onto the degenerate subspace and then normalized.) Thus if the measurement result is the non-degenerate eigenvalue λj then the state |ψ⟩ collapses to P |ψ⟩ |ψ⟩ → √ j , ⟨ψ|Pj|ψ⟩ where the square root in the denominator simply normalizes the col- lapsed state. Introduce this projector idea for measurements to avoid the confusion that some students think measurement of X is repre- sented by multiplying by X. Discuss further the confusion that Pauli operators are both unitary operations and Hermitian observables. If the Stern-Gerlach magnet measures ⃗σ · nˆ (i.e. asks ‘Are you in state | +n ˆ⟩ or state | − nˆ⟩?), then from the Born rule, the probability that the measurement result is ±1 is |⟨±nˆ|ψ⟩|2. If you make a measurement of a product state, say | ↑↑⟩ that asks, ‘Which entangled Bell state are you in?’ the state will collapse to one of the Bell states and the collapse (aka ‘measurement back action’) leaves the state entangled. Chapter 4

Quantum Error Correction

Now that we understand entanglement, we are in a position to tackle quan- tum error correction. To overcome the deleterious effects of electrical noise, cosmic rays and other hazards, modern digital computers rely heavily on error correcting codes to store and correctly retrieve vast quantities of data. Classical error correction works by introducing extra bits which provide redundant encoding of the information. Error correction proceeds by measuring the bits and comparing them to the redundant information in the auxiliary bits. Another benefit of the representation of information as discrete bits (with 0 and 1 corresponding to a voltage for example) is that one can ignore small noise voltages. That is, V = 0.99 volts can be safely assumed to represent 1 and not 0. All classical (and quantum) error correction codes are based on the as- sumption that the hardware is good enough that errors are rare. The goal is to make them even rarer. For classical bits there is only one kind of error, namely the bit flip which maps 0 to 1 or vice versa. We will assume for simplicity that there is probability p ≪ 1 that a bit flip error occurs, and that error occurrences are uncorrelated among different bits. One of the sim- plest classical error correction codes to understand involves repetition and majority rule. Suppose we have a classical bit carrying the information we wish to protect from error and we have available two ancilla bits (also sub- ject to errors). The procedure consists copying the state of the first bit into the two ancilla bits. Thus a ‘logical’ 1 is represented by three ‘physical’ bits in state 111, and a ‘logical’ 0 is represented by three ‘physical’ bits in state 000. Any other physical state is an error state outside of the logical state

63 CHAPTER 4. QUANTUM ERROR CORRECTION 64 space. Suppose now that one of the three physical bits suffers an error. By examining the state of each bit it is a simple matter to identify the bit which has flipped and is not in agreement with the ‘majority.’ We then simply flip the minority bit so that it again agrees with the majority. This procedure succeeds if the number of errors is zero or one, but it fails if there is more than one error. Of course since we have replaced one imperfect bit with three imperfect bits, this means that the probability of an error occurring has increased considerably. For three bits the probability Pn of n errors is given by 3 P0 = (1 − p) (4.1) 2 P1 = 3p(1 − p) (4.2) 2 P2 = 3p (1 − p) (4.3) 3 P3 = p . (4.4) Because our error correction code only fails for two or more physical bit errors the error probability for our logical qubit is 2 3 plogical = P2 + P3 = 3p − 2p , (4.5) If p < 1/2, then the error correction scheme reduces the error rate (instead of −6 −12 making it worse). If for example p = 10 , then plogical ∼ 3×10 . Thus the lower the raw error rate, the greater the improvement. Note however that even at this low error rate, a petabyte (8 × 1015 bit) storage system would have on average 24,000 errors. Futhermore, one would have to buy three petabytes of storage since 2/3 of the disk would be taken up with ancilla bits! We are now ready to enter the remarkable and magic world of quantum error correction. Without quantum error correction, quantum computation would be impossible and there is a sense in which the fact that error cor- rection is possible is even more amazing and counterintuitive than the fact of quantum computation itself. Naively, it would seem that quantum error correction is completely impossible. The no-cloning theorem (see Box 2.3) does not allow us to copy an unknown state of a qubit onto ancilla qubits. Furthermore, in order to determine if an error has occurred, we would have to make a measurement, and the back action (state collapse) from that mea- surement would itself produce random unrecoverable errors. Part of the power of a quantum computer derives from its analog character–quantum states are described by continuous real (or complex) vari- ables. This raises the specter that noise and small errors will destroy the CHAPTER 4. QUANTUM ERROR CORRECTION 65 extra power of the computer just as it does for classical analog computers. Remarkably, this is not the case! This is because the quantum computer also has characteristics that are digital. Recall that any measurement of the state of qubit always yields a binary result. Because of this, quantum errors are continuous, but measured quantum errors are discrete. Amazingly this makes it possible to perform quantum error correction and keep the calcula- tion running even on an imperfect and noisy computer. In many ways, this discovery by Peter Shor in 1995 and Andrew Steane in 1996 is even more profound and unexpected than the discovery of efficient quantum algorithms that work on ideal computers. It would seem obvious that quantum error correction is impossible be- cause the act of measurement to check if there is an error would collapse the state, destroying any possible information. Remark- ably however, one can encode the information in such a way that the presence of an error can be detected by measurement, and if the code is sufficiently sophisticated, the error can be corrected, just as in classical computation. Classically, the only error that exists is the bit flip. Quantum mechani- cally there are other types of errors (e.g. phase flip, energy decay, erasure channels, etc.). However codes have been developed (using a minimum of 5 qubits) which will correct all possible quantum errors. By concatenating these codes to higher levels of redundancy, even small imperfections in the error correction process itself can be corrected. Thus quantum superposi- tions can in principle be made to last essentially forever even in an imperfect noisy system. It is this remarkable insight that makes quantum computation possible. As an entr´eto this rich field, we will consider a simplified example of one qubit in some state α|0⟩+β|1⟩ plus two ancillary qubits in state |0⟩ which we would like to use to protect the quantum information in the first qubit. As already noted, the simplest classical error correction code simply replicates the first bit twice and then uses majority voting to correct for (single) bit flip errors. This procedure fails in the quantum case because the no-cloning theorem (see Box 2.3) prevents replication of an unknown qubit state. Thus there does not exist a unitary transformation which takes

[α|0⟩ + β|1⟩] ⊗ |00⟩ −→ [α|0⟩ + β|1⟩]⊗3. (4.6)

As was mentioned earlier, this is clear from the fact that the above transfor- mation is not linear in the amplitudes α and β and quantum mechanics is CHAPTER 4. QUANTUM ERROR CORRECTION 66 linear. One can however perform the repetition code transformation:

[α|0⟩ + β|1⟩] ⊗ |00⟩ −→ [α|000⟩ + β|111⟩], (4.7) since this is in fact a unitary transformation. Just as in the classical case, these three physical qubits form a single logical qubit. The two logical basis states are

|0⟩log = |000⟩

|1⟩log = |111⟩. (4.8) The analog of the single-qubit Pauli operators for this logical qubit are readily seen to be

Xlog = X1X2X3

Ylog = iXlogZlog

Zlog = Z1Z2Z3. (4.9) We see that this encoding complicates things considerably because now to do even a simple single logical qubit rotation we have to perform some rather non-trivial three-qubit joint operations. It is not always easy to achieve an effective Hamiltonian that can produce such joint operations, but this is an essential price we must pay in order to carry out quantum error correction. It turns out that this simple code cannot correct all possible quantum errors, but only a single type. For specificity, let us take the error operating on our system to be a single bit flip, either X1,X2, or X3. These three together with the identity operator, I, constitute the set of operators that produce the four possible error states of the system we will be able to correctly deal with. Following the formalism developed by Daniel Gottesman, let us define two stabilizer operators

S1 = Z1Z2 (4.10)

S2 = Z2Z3. (4.11) These have the nice property that they commute both with each other (i.e., [S1,S2] = S1S2 − S2S1 = 0) and with all three of the logical qubit operators listed in Eq. (4.9). This means that they can both be measured simulta- neously and that the act of measurement does not destroy the quantum information stored in any superposition of the two logical qubit states. Fur- thermore they each commute or anticommute with the four error operators CHAPTER 4. QUANTUM ERROR CORRECTION 67 in such a way that we can uniquely identify what error (if any) has occurred. Each of the four possible error states (including no error) is an eigenstate of both stabilizers with the eigenvalues listed in the table below

error S1 S2 I +1 +1 X1 −1 +1 X2 −1 −1 X3 +1 −1 Thus measurement of the two stabilizers yields two bits of classical informa- tion (called the ‘error syndrome’) which uniquely identify which of the four possible error states the system is in and allows the experimenter to correct the situation by applying the appropriate error operator, I,X1,X2,X3 to the system to cancel the original error. We now have our first taste of fantastic power of quantum error correction. We have however glossed over some important details by assuming that either an error has occurred or it hasn’t (that is, we have been assuming we are in a definite error state). At the next level of sophistication we have to recognize that we need to be able to handle the possibility of a quantum superposition of an error and no error. After all, in a system described by smoothly evolving superposition amplitudes, errors can develop continuously. Suppose for example that the correct state of the three physical qubits is

|Ψ0⟩ = α|000⟩ + β|111⟩, (4.12) and that there is some perturbation to the Hamiltonian such that after some time there is a small amplitude ϵ that error X2 has occurred. Then the state of the system is √ 2 |Ψ⟩ = [ 1 − |ϵ| I + ϵX2]|Ψ0⟩. (4.13) (The reader may find it instructive to verify that the normalization is correct.) What happens if we apply our error correction scheme to this state? The measurement of each stabilizer will always yield a binary result, thus illustrat- ing the dual digital/analog nature of quantum information processing. With 2 probability P0 = 1 − |ϵ| , the measurement result will be S1 = S2 = +1. In this case the state collapses back to the original ideal one and the error is removed! Indeed, the experimenter has no idea whether ϵ had ever even developed a non-zero value. All she knows is that if there was an error, it is CHAPTER 4. QUANTUM ERROR CORRECTION 68 now gone. This is the essence of the Zeno effect in quantum mechanics that repeated observation can stop dynamical evolution. (It is also, once again, a clear illustration of the maxim that in quantum mechanics ‘You get what you 2 see.’) Rarely however (with probability P1 = |ϵ| ) the measurement result will be S1 = S2 = −1 heralding the presence of an X2 error. The correction protocol then proceeds as originally described above. Thus error correction still works for superpositions of no error and one error. A simple extension of this argument shows that it works for an arbitrary superposition of all four error states.

4.1 An advanced topic for the experts

Not covered in 2019. There remains however one more level of subtlety we have been ignoring. The above discussion assumed a classical noise source modulating the Hamiltonian parameters. However in reality, a typical source of error is that one of the physical qubits becomes entangled with its envi- ronment. We generally have no access to the bath degrees of freedom and so for all intents and purposes, we can out the bath and work with the reduced of the logical qubit. Clearly this is generically not a pure state. How can we possibly go from an impure state (containing the entropy of entanglement with the bath) to the desired pure (zero entropy) state? Ordinary unitary operations on the logical qubit preserve the entropy so clearly will not work. Fortunately our error correction protocol involves applying one of four possible unitary operations conditioned on the outcome of the measurement of the stabilizers. The collapse associated with the measurement gives us just the non-unitarity we need and the error correction protocol works even in this case. Effectively we have a Maxwell demon which uses Shannon information entropy (from the measurement re- sults) to remove an equivalent amount of from the logical qubit! To see that the protocol still works, we generalize Eq. (4.13) to include the bath √ 2 |Ψ⟩ = [ 1 − |ϵ| |Ψ0, Bath0⟩ + ϵX2]|Ψ0, Bath2⟩. (4.14) For example, the error could be caused by the second qubit having a coupling to a bath operator O2 of the form

V2 = g X2O2, (4.15) CHAPTER 4. QUANTUM ERROR CORRECTION 69 acting for a short time ϵ~/g so that

|Bath2⟩ ≈ O2|Bath0⟩. (4.16)

Notice that once the stabilizers have been measured, then either the exper- imenter obtained the result S1 = S2 = +1 and the state of the system plus bath collapses to

|Ψ⟩ = |Ψ0, Bath0⟩, (4.17) or the experimenter obtained the result S1 = S2 = −1 and the state collapses to

|Ψ⟩ = X2|Ψ0, Bath2⟩. (4.18)

Both results yield a product state in which the logical qubit is unentangled with the bath. Hence the algorithm can simply proceed as before and will work. Finally, there is one more twist in this plot. We have so far described a measurement-based protocol for removing the entropy associated with errors. There exists another route to the same goal in which purely unitary multi- qubit operations are used to move the entropy from the logical qubit to some ancillae, and then the ancillae are reset to the ground state to remove the entropy. The reset operation could consist, for example, of putting the ancillae in contact with a cold bath and allowing the qubits to spontaneously and irreversibly decay into the bath. Because the ancillae are in a mixed state with some probability to be in the excited state and some to be in the ground state, the bath ends up in a mixed state containing (or not containing) photons resulting from the decay. Thus the entropy ends up in the bath. It is important for this process to work that the bath be cold so that the qubits always relax to the ground state and are never driven to the excited state. We could if we wished, measure the state of the bath and determine which error (if any) occurred, but in this protocol, no actions conditioned on the outcome of such a measurement are required. Quantum error correction is extremely challenging to carry out in prac- tice. In fact the first error correction protocol to actually reach the break even point (where carrying out the protocol extends rather than shortens the lifetime of the quantum information) was achieved by the Yale group in 2016. This was done not using two-level systems as qubits but rather by CHAPTER 4. QUANTUM ERROR CORRECTION 70 storing the quantum information in the states of a harmonic oscillator (a superconducting microwave resonator containing superpositions of 0, 1, 2,... photons). Chapter 5

Yet To Do

1. Insert 2019 homework as exercises in text

2. Did I insert proof that length preservation implies general inner product conservation which implies unitary? Proof could be improved.

3. Appendix on linear algebra, outer products, direct sums, eigenval- ues, degeneracies, determinants, traces, hermitian matrices producing bases, etc.

4. 3 CNOTS make a SWAP but apply to a general state or to the X basis

5. effect of Hadamards on CNOT

6. quantum teleportation

7. Shor code as next step beyond 3 qubit repetition code

8. for harmonic oscillator?

71 Bibliography

[1] Bennett, Charles H. and Wiesner, Stephen J., ‘Communication via one- and two-particle operators on Einstein-Podolsky-Rosen states,’ Phys. Rev. Lett. 69, 2881-2884 (1992).

[2] A. Einstein and B. Podolsky and N. Rosen, ‘Can quantum-mechanical description of physical reality be considered complete?’, Phys. Rev. 47, 777-780 (1935).

[3] J. S. Bell, ‘On the Einstein Podolsky Rosen paradox,’ Physics 1, 195–200 (1964).

[4] J. F. Clauser, M. A. Horne, A. Shimony, and R. A. Holt, ‘Proposed experiment to test local hidden-variable theories,’ Phys. Rev. Lett. 23, 880–884, (1969).

[5] C. H. Bennett, G. Brassard, C. Cr´epeau, R. Jozsa, A. Peres, and W. K. Wootters, ‘Teleporting an unknown quantum state via dual classical and Einstein-Podolsky-Rosen channels,’ Phys. Rev. Lett. 70, 1895–1899 (1993).

72